Avatar

Canonical Momentum [highlights]

@dldqdot / dldqdot.tumblr.com

A blog to collect canonicalmomentum's longer posts
Avatar
reblogged
Avatar
canmom

Designing a spaceship with spin gravity - part 1

I’m working on designing a hard-ish sci-fi spaceship for a project with @mxmy. It’s a generation ship, but the journey time is reduced by some kind of magic thing to make wormholes, and the crew live inside spinning ring sections which provide gravity while the ship is not under acceleration.

By hard-ish, I mean I’d like it to seem ‘realistic’ to someone who has some idea how spaceships work, but not have everything calculated in full engineering detail, and allow some black-box-y future tech.

The builders of the spaceship are a future society of humans who live on Mars. Previous ships have travelled to other parts of the solar system, and this is the first one to go interstellar. It is, I imagine, a huge investment for the people who built it.

The destination is, if I recall right, the planet we currently call Kepler-186f, which is 558ly away - a huge distance, which makes it lucky they have the wormhole thingy.

The magic wormhole thingy can create a wormhole connecting a point ahead of the ship to a certain distance away, determined by how well it can be imaged by the ship’s instruments. This will allow, I presume, FTL. I don’t know all the details of this thing. (Heck, Maki just kinda vaguely said ‘space-folding’, but the way she described it sounded wormholey so I’m using that language). Not sure the targeted journey time, but with the wormhole thing we can probably finagle things pretty arbitrarily.

The ship otherwise accelerates like an ordinary spaceship, with some kinda rocket engine. Is it fusion? Antimatter? I don’t know, and it may not have been decided!

This post is mostly worrying about the spin gravity.

Turn those spin sections

Here’s a very rough not-to-scale initial design for such a spaceship. Engine at the back (it would need to be far enough away to not irradiate everything, since any interstellar engine is probably radioactive), some kind of instruments at the front (also maybe the wormhole machine), and a middle section with two counter-rotating spin sections on spokes. (I was told there’d be three spin sections, but for now I’m drawing two since it nicely balances angular momentum).

(apologies for the video quality and lack of looping. It does not appear to be possible to embed a Tumblr video in a text post, so we’re stuck with a suboptimal Youtube video. I rendered this video too small for Youtube to go above 360p, so it’s kinda blurry!)

There are various potential problems with such a design:

  • how to spin the spin sections up in the first place.
  • Since we have two spin sections, they can be spun up simultaneously without causing the central hub in the opposite direction. Still, we need some means to accelerate the spin sections up to the desired rotational speed.
  • friction where the spin sections meet the central axle.
  • With a single spin section, this will gradually cause the central hub to spin up and the spin sections to spin down (to conserve angular momentum) until they’re both spinning at the same rate. If both sections have equal friction at the hub they’ll apply equal torque and the central hub won’t spin up, but the spin sections will still slowly lose rotational kinetic energy to heat until they’re no longer spinning.
  • getting people and material between spin sections and into the central axle.
  • This requires some kind of coupling through which people and air can pass through.

Spinning the spin sections up

The initial spin up can be achieved either using electromagnetic acceleration (similar to a maglev train) or using small rockets. Small rockets are limited by the amount of propellent, electromagnetic acceleration requires generation of strong electric currents.

To maximise torque, forces should be applied at the outer edge of the spin section rather than at the centre. This is easy for rockets: they can just be mounted on the outside of the spin sections.

For electromagnetic acceleration, the sections migh be spun up during construction (some kind of cradle), or else you might have a stationary boom holding EM acceleration devices like so:

In this picture, the yellow parts are supporting EM acceleration thingies at four points in the ring, and each ring has some kind of track running around it (also yellow) that responds to the accelerators. By applying electromagnetic forces through these accelerators, the wheels can be spun up. By accelerating both wheels at once, the central axle remains unspinning.

Why have this arrangement? It may be useful to adjust the spin of the wheels during flight, in order to e.g. compensate for friction losses, adjust the gravity in the ring, or store energy in the rotating wheels (like giant flywheels).

Frictionless bearings?

Unless the point where the rings join the axle is nigh frictionless, the rings will spin down over the flight.

One way to make a frictionless bearing might be to also make it magnetic - essentially each ring is something like a maglev train, or in another view, magnetic forces keep the axle from getting too close to the inside of the wheel hub. How hard would this be to engineer? I don’t know. The advantage is that there is no contact at all between the spin sections and the axle. A disadvantage is that the system only works where electricity is flowing, so it is a drain on the ship’s power systems.

I worry in such a configuration that when the ship is under acceleration, the electromagnetic bearings would not be able to hold the wheel in place. Perhaps the acceleration booms could also help keep the wheels spinning in place…

More of a problem is transforming people and materials between the rings and the hub.

It might be possible to generate electricity and carry water etc. inside the rings themselves without having anything pass anything out from the bearings. However, the greater the mass in the ring sections, the greater the moment of inertia of the rings - which means they would take more energy to spin up or down. So having heavy power plants or water tanks out in the rings may be a problem.

That said, having a water buffer around the edges of the ring sections would be a useful protection against space radiation. I suppose it may also be possible to transfer power using electromagnetic radiation to the spin sections, allowing electricity to be generated on the central axle and transferred to the rings without requiring contact.

This suggests a design where the ring sections are each nearly entirely sealed environments, with the only thing coming from outside being electrical energy. Food is grown, water is stored, used, cleaned and recycled, materials are processed and used entirely within the ring. The living sections are completely surrounded by tanks of water which absorb radiation from outside.

This is all great… until you want to get from one ring section to the other. That requires either a pressurised route from one section to the other, or a pressurised container - a capsule or a spacesuit - to crawl down one of the spokes, transfer to the other ring’s spokes, and crawl back out.

That doesn’t sound too unreasonable actually. Let’s add some little cars to our picture.

So in this arrangement, the car (strong green) crawls down the track on the inside of a spoke, waits until it’s lined up with a track on the axle, jumps over to the axle track, crawls over to the other hub, jumps over to the spoke track, and crawls back up to the outside of the other axle. It will be moving relative to the target track at each ‘jump’, so there will have to be some means of ‘catching’ it.

This will involve a few transfers of angular momentum, but because the car is much lighter than the rings, relatively minor ones.

Now I’m kind of wondering though… apart from the ‘car’ is there any reason to have the ‘spokes’ at all? Could the ‘wheels’ be entirely supported at the rim?

But as long as there’s reason to go down up to the centre, I guess we should stick to this arrangement.

As for what it’s like if you’re a passenger… Suppose you live in the aft ring. The outer surface of the ring is ‘down’, and the inner is ‘up’. If you want to go visit the centre or the fore ring, you climb up into a little car thingy. The car rises ‘up’, and as it gets closer to the centre, the ‘gravity’ you feel goes down to almost nothing. At the centre there’s a ‘clunk’ as the track passes you over to a suitable cradle on the hub. What little ‘gravity’ remains disappears: you are (assuming the ship is not under acceleration) in freefall.

What else is missing?

So without getting into the details of what happens inside the spin sections, we have the basic outlines of a spaceship.

But there are still important parts this ship is missing. In particular, there is nowhere to store reaction mass, and no way for the heat generated by the engine and living sections to be safely radiated into space.

Let’s add some stuff to the engine area. Big chunky reaction mass/fuel tanks, and some heat radiators.

I have no idea if the remass tanks or radiators are the right size, but then, this diagram has never been attempting to be to scale, and getting them right would require more effort at calculation. Assuming the material in the tanks is of a comparable density to the rest of the ship, and judging by eye that they’re of similar volumes this means it has a mass ratio of only about 2(ish). That’s not going to be a huge amount of delta-v.

Also, hold on a minute, wouldn’t the radiation from engine irradiate these fuel tanks? And also probably the crew rings The ship may need to be a lot longer and the tanks a lot narrower to employ a shadow shield, similar to the NASA Arthur C Clark design. I’ve also moved the angular magnetic accelerators to be on the aft side of the rings, since this would hopefully help transfer force to the rings when the ship is under acceleration.

Still this is probably a decent starting point. Here’s another spinaround with all the various additions.

Next update we’ll see about getting everything safely into the radiation shadow of the engines, and possibly eliminating the wheel hubs?

Comments welcome! @erin-space-goat and @bloodthreadsaltglassandtears you might have some ideas how to develop this further?

Avatar
reblogged
Avatar
canmom

How to fly a rocket (in space!)

This is the first post in a series to try and cover all the main concepts of flying rockets around in space in a reasonably accessible way.

I am, sadly, not an astronaut, but I’ve done most of a physics degree, and Kerbal Space Program has convinced me that rocketry can actually be really simple and intuitive. Hopefully these posts will be helpful if you want to write about science fiction spaceships or the like.

There is a small handful of equations. I’m using MathJax, so click through to my blog if you want to see them properly.

Why are rockets a thing?

On Earth, if we want to move somewhere, we can do so by pushing off something else: the ground (for walkers, cars, etc.) or the air (for aeroplanes, birds, etc.). In space, there is nothing to push against.

There are, basically, two options. Either you can push against what little there is (such as sunlight, or a huge laser array back home), or you can shoot part of your spaceship away from the rest of it, causing the remaining parts of the spaceship to move in the opposite direction. All space engines, not matter how complicated, amount to doing one of these two things, usually the latter.

Here’s a very simple illustration “rocket”: the arm drive. An astronaut picks up bowling balls in throws them out the back of her rocket. This makes the rocket go forwards.

Jargon: the stuff that you throw out the back of your rocket is called reaction mass. This is distinct from fuel which provides the energy, though in the case of chemical rockets, the fuel and reaction mass are one and the same.

You use a rocket to change your velocity. Velocity is a thing that combines speed and direction: an object going 5 metres per second north, and an object going five metres per second west have the same speed but different velocities.

The good news about being in space is that nothing is going to slow you down. On Earth, friction and air resistance always conspire to make anything moving relative to the surface of the Earth, or at least to the air in the case of an aeroplane, slow down and stop. In space, there is nothing. So whatever your velocity is, it will stay the same unless affected by gravity.

So a rocket consists of two parts: the thing that you actually care about (sometimes called the payload) plus the structure of the rocket, and ‘reaction mass’ which will be launched out of the rocket to change its velocity.

Mass ratios and the almighty rocket equation

OK, here’s where it gets sticky. When you turn on the rocket, it doesn’t just have to move the rocket, but also all the reaction mass you’re still holding on board to throw out later. So the amount of reaction mass you need to change your velocity by a certain amount increases much faster than the velocity change you want to make.

Some more jargon: turning on your rocket to change your velocity is called a burn. The ratio of the mass of the rocket at the beginning of the burn \(M_i\) to the mass ratio at the end of the burn \(M_f\) (after losing some reaction mass) is called the mass ratio. The change in the velocity of your rocket is called the delta-v (“delta vee”), or \(\Delta v\).

(Reason for the name: the greek letter Delta \(\Delta\) represents a change in something: \(\Delta v\) is “change in \(v\)”. \(v\) is the usual symbol for velocity.)

There’s an equation called the Tsiolkovsky Rocket Equation which says that the change in the velocity of a rocket after a burn depends on the exhaust velocity \(v_e\) and the mass ratio \(M_i/M_f\) according to $$\Delta v = v_e \log \frac{M_i}{M_f}$$What this means is that the mass ratio is exponential in the delta-v: if you want to change your velocity by twice the amount, you need the mass ratio to increase by a factor of 7.4; if you want to change your velocity by three times the amount you need to increase your mass ratio by a factor of 20.1; it quickly becomes ridiculous.

If you’d like to know more about the maths, I work through the derivation of the Tsiolkovsky rocket equation and its relativistic counterpart here.

Practically speaking, only so much of your rocket can consist of reaction mass. The Saturn V used in the Apollo moon missions had a mass ratio of about 60, meaning 60 times the mass of the payload (the various modules used at the moon) was expelled either as reaction mass or as spent stages. In other words, the vast majority of material in your rocket is not actually useful, but just going to be thrown out the back.

This also places harsh limits on the amount of mass you can carry. Every kilogram of stuff you carry in your rocket requires you to carry tens or hundreds of kilograms of reaction mass to get the same delta-v. Rockets must be as light as possible and there’s no real way around that.

The two important numbers: thrust & specific impulse

To improve the amount of velocity change you can get from a given mass ratio, the only thing you can do is to increase the exhaust velocity, also known as your specific impulse (a name coming from the fact that it’s the force divided by the mass flow rate, 'specific’ being used by scientists to mean 'per unit mass’).

A chemical rocket, though easier to build than other kinds of space engine, has a relatively low exhaust velocity, around \(4,000\unit{m s^{-1}}\). Other propulsion systems, such as ion drives, can achieve much higher specific impulses; Wikipedia offers around \(30,000\unit{m s^{-1}}\) for a typical ion thruster, or more for more speculative designs such as the VASIMR.

The drawback of ion thrusters, and some other high specific impulse propulsion systems, is that they have very low thrust. Thrust is the other measure of a rocket: it’s just how much force a rocket applies to the ship when it is running, and hence how quickly it changes the velocity (the acceleration - note unlike everyday use of 'acceleration’, to physicists 'acceleration’ includes both changes of speed and direction). A low thrust rocket can achieve the same changes in velocity as a high thrust rocket, but it takes much longer to do it.

So in general the change of velocity in a burn depends on the thrust, mass of the rocket, and the duration of the burn.

In the distant future, much more speculative designs exist such as fusion rockets that have both high thrust and high specific impulse. But presently we are forced to choose.

High thrust is very important when leaving the ground. But we will come to that later.

Spaceships have their engines turned off most of the time

Because mass ratios increase so quickly, rockets can only change their velocity by so much before they expend all their reaction mass. This is called the delta-v budget of a mission.

Following the metaphor, a spaceship will 'spend’ its delta-v budget to perform maneuvers. Maneuvers - such as changing from one orbit to another - have a certain delta-v 'cost’.

The rest of the time, the rocket is inactive. Generally the rockets will only burn for a tiny portion of the mission.

This is unintuitive, since we’re used to Earth vehicles which are constantly losing energy to friction and air resistance, and therefore must have their engines running all the time. Sci-fi films frequently decide they’d prefer to show spaceships with the engines glowing, even if the ship is not performing a maneuver.

(Low-thrust engines such as ion drives are a semi-exception, in that they must burn for much longer than high-thrust engines to complete maneuvers, as mentioned in the previous section. But they still only activate their engines when they are changing their velocity.)

Orbits

Usually your rocket is not just hanging out in empty space; there is something big and heavy such as a planet or star in the vicinity that will change its trajectory through gravity. So lets talk about orbits.

When there are lots of planets, moons, etc. around it gets quite complicated, but we can start from building up from the simplest case: when your rocket is orbiting just one thing.

There are basically two kinds of orbits we’re concerned about: elliptical orbits and hyperbolic orbits. There is a third kind, parabolic orbits, which we won’t worry about because they only occur in ridiculously specific circumstances.

Elliptical orbits (space eggs)

Elliptical orbits are what you’d generally think of as an orbit, as in, your rocket’s going round a thing. They look like this:

Mathematically, this is an ellipse with one of the two foci (singular 'focus’) on the centre of the massive object (planet, star, etc.). For simplicity’s sake, we’ll call the thing it’s orbiting a planet.

Elliptical orbits are characterised by a parameter called the eccentricity, which is usually written \(e\). The eccentricity tells you how how long and narrow the ellipse is. A circular orbit has an eccentricity of zero. An orbit with a higher eccentricity moves further away from the thing it’s orbiting, and back, over time. Elliptical orbits all have eccentricities between zero and 1.

There are two important points on an orbit: the periapsis and the apoapsis. (These have special names in some cases: for example if something is orbiting Earth we call them the perigee and apogee, if something’s orbiting the sun the perihelion and aphelion, etc.). The periapsis is the point on the orbit closest to the planetg. The apoapsis is the point furthest away.

The second important parameter is the semi-major axis, usually written (a). This is just how big the orbit is: it’s half the length between the periapsis and apoapsis. For a circular orbit, the semi-major axis is the radius of the circle.

The periapsis is a distance \((1-e)a\) from the centre of the planet. The apoapsis is a distance \((1+e)a\) away.

The velocity of a rocket on an elliptical orbit is constantly changing, but its direction is always a tangent of the ellipse. And now an important point: the speed of a rocket is faster when it’s near the planet, and slower when it’s further from the planet. As the rocket comes in towards the periapsis, it picks up speed; as it moves out towards the apoapsis, it loses speed. This means the rocket spends a lot more time in the further away parts of the orbit than the nearby parts of the orbit. (The exact speed it has is precisely determined by for example the vis-viva equation, but I’m trying to avoid giving too much detail here.)

Here’s an animation to illustrate this principle:

[modified from this animation by Brandir on Wikimedia Commons; license CC BY-SA 3.0.]

Which orbit does a rocket actually follow? It depends on its position and velocity: every combination of position and velocity corresponds to a different orbit. For example, here’s a picture of a succession of orbits that each pass through the same point in the same direction, but with different speeds. For some of these orbits, this point is the apoapsis; for others it is the periapsis. Dividing these two groups of orbits is a circular orbit. There are two orbits shown for each eccentricity, with different semi-major axes.

So far, we’ve been showing two-dimensional representations of orbits, all in the same plane. However, an orbit can be in any plane containing the central planet.

Astronomers have a number of different ways to describe orbits. In general, you need five numbers to pick out a specific orbit, and a sixth number to identify a position on that orbit. One way is to pick a position (three coordinates) and velocity (three components). Another important way is the Keplerian orbital elements.

Hyperbolic orbits trajectories

Hyperbolic ‘orbits’, or if you prefer hyperolic trajectories, are what happens when your spaceship is going too fast to stay in orbit. This is the course you follow if you’re escaping from a planet’s gravity, or if you’re doing a flyby.

A hyperbolic orbit looks like this:

Mathematically, this is one of the two branches of a hyberbola. Like with the ellipse, its focus is on the centre of the planet. It has a periapsis, but no apoapsis. Many of the same rules apply: the spaceship gets faster as it gets closer to the periapsis, and slows down as it moves away.

As the ship moves further and further away from the planet in its hyperbolic orbit, it gets closer and closer (asymptotically) to moving on a straight line. This makes intuitive sense: in the absence of a planet, the ship would move in a straight line, and the planet’s gravitational influence gets less and less significant the further away you travel.

Like ellipses, hyperbolas have a parameter called the eccentricity \(e\). While for an ellipse, the eccentricity is always greater than or equal to zero, and less than 1, for a hyperbola it can be any number greater than 1. The reason both kinds of curves have this parameter is because hyperbolas and ellipses are both members of a family of curves called conic sections. (The parabolic trajectory, which we mentioned above but won’t discuss in detail, has an eccentricity of exactly 1. It’s nigh impossible to get onto a parabolic orbit in real life, and it’s pretty much the same as a hyperbolic orbit so we won’t worry about it.)

Here’s a quick summary of possible cases for the eccentricity:

  • \(e=0\): circular orbit
  • \(0<e<1\): elliptical orbit
  • \(e=1\): parabolic trajectory
  • \(e>1\): hyperbolic trajectory

For a hyperbola, the eccentricity basically measures how much the ship’s path curves around the planet. A spaceship following a hyperbola with a high eccentricity barely has its trajectory changed by the encounter with the planet. One with a low eccentricity can have its trajectory changed dramatically.

Like with an ellipse, the hyperbola also has a parameter called \(a\), again called the semi-major axis, though the reason for that name is less obvious. The periapsis is a distance \((e-1)a\) from the focus.

Here are some hyperbolic trajectories through a point:

The eccentricity can go as high as you like. The higher it goes, the closer it gets to a straight line. This corresponds to going very quickly past something very light that barely affects your course.

Energy & angular momentum: the unchanging numbers

Here’s a more mathematical bit.

So one important thing about orbits round a single thing like this is that there a two special things you can calculate at any point on your orbit, and you will get the same answer every time. These are the orbital energy and the angular momentum. Physicists will call these numbers conserved quantities.

These unchanging numbers have to do with the symmetry of the situation, which is something I will talk about in another post on Noether’s theorem at some point, eventually (if I keep promising to write this post, maybe it willl happen!)

Here’s the orbital energy $$E=-\frac{GMm}{r}+\frac{1}{2}mv^2$$where \(M\) is the mass of the thing you’re orbiting (keep calling it a planet), \(m\) is the mass of your rocket, \(r\) is your distance from the planet at a given moment, and \(v\) is your velocity at the same moment.

The first term is called the potential energy, and it gets more and more negative as you get closer to the planet. The second term is the kinetic energy and it gets bigger as you go faster. So as you go closer to the planet, your potential energy gets more negative, but your speed goes up by an amount that exactly counters that change so overall the total energy stays the same.

Divide out the mass of your rocket and you have the specific orbital energy (there’s that word specific meaning ‘per unit mass’ again). So the specific orbital energy is $$e=-\frac{GM}{r}+\frac{1}{2}v^2$$This is a handy quantity because it’s the same for anything on a given orbit, no matter how heavy the orbiting object.

The specific orbital energy is important because you can relate it to the size of the orbit, according to $$e=-\frac{GM}{2a}$$ for an ellipse, and $$e=\frac{GM}{2a}$$ for a hyperbola, where \(a\) is the semi-major axis in both cases. (This means an elliptical orbit has positive specific orbital energy, and a hyperbolic orbit has negative specific orbital energy). So an orbit with a lower specific orbital energy is smaller than one with a higher spefici orbital energy.

The orbital angular momentum is $$L=\frac{1}{2}mr\dot{\phi}^2$$ where \(\dot{\phi}\) is the rate of change of the angular coordinate, basically a measure of how quickly you’re going around the planet without regard for how much you’re going towards or away from the planet. So as you get closer to the planet, you don’t just need to be going faster in general, you need to be going faster around the planet. But we won’t talk about this one so much in this post.

The point of this is: these numbers are unchanging over a given orbit. So you can think of rocketry maneuvers and other things that affect the speed of your rocket changing these numbers, and therefore changing the orbit you’re on.

If you reduce your orbital energy, you will generally be put on a tighter orbit that goes closer to the planet.

For example: if you are flying through a thin atmosphere, you will lose orbital energy. This will put you onto a smaller, tighter orbit with a lower orbital energy. You might quickly regain the speed you lost as you move down to a lower orbit, but you’ve lost orbital energy. Since the orbital energy is unchanging under most circumstances, it’s easier to talk about that than your speed, which is constantly going up and down.

Six directions

Right, here are the things your rocket might do.

First of all we’re going to have to name some directions! The “mathematical” names are the terms used in the Frenet-Serret formulas. The other names are the ones used in Kerbal Space Program.

Prograde (mathematically, tangential) is the direction your rocket is going in in its orbit.

Retrograde (mathematically anti-tangential I guess) is opposite the direction your rocket is going in.

Radial and anti-radial, or radial-in and radial-out (mathemtatically, normal and anti-normal), are perpendicular to the direction of your orbit, in the plane of the orbit. It’s a matter of convention whether “radial” is inwards (roughly towards the planet) or outwards (roughly away from the planet). Kerbal Space Program uses the convention that “radial” is inwards. For clarity, I’m going to stick to “radial-in” and “radial-out”.

Normal and anti-normal (mathematically binormal and anti-binormal) are perpendicular to the plane of the orbit.

And yes, the makers of KSP decided to use a different meaning of the word “normal” to the Frenet-Serret thing.

These are independent of which way your rocket is pointing - they depend only on where you are, where the centre of the planet is, and which way you’re going.

Your rocket can in fact point in any direction. It does not have to fly “forwards” because there is no atmosphere to worry about streamlining. This is quite different from aircraft on Earth - an aircraft can fly in a way that’s not pointing straight forward, but it’s not usually a good situation to be in. A rocket, on the other hand, needs to point whichever direction is appropriate for the next maneuver, or to expose its solar panels to the sun, or spin to provide a sensation of “gravity” to the astronauts inside - all sorts of possibilities.

Since it can point in any direction, it can burn in any direction. But any burn can be broken into ‘components’ along these six directions. So we can talk about each of them individually.

The next post

We’re going to go into detail about the different directions you might want to burn in, and what they do to your orbit. And we’ll use that to talk about important maneuvers like circularisation burns, Hohmann transfer orbits, and how you go about making a rendezvous with another spaceship!

(And if you’re going to the Nine Worlds convention in London this year, I’m running a panel on rockets and rocket design with @darmokontheocean and @clockworkpipedreams covering basically the same ground as this post.)

Avatar
reblogged
Avatar
canmom

I just had an idea: a maglev mass driver for space launch that's similar to the "StarTram", but accelerating in a circular loop similar to the LHC. What I want to know: how big a loop would I need to hit 10 km/s speed while keeping rcf under 4g?

Avatar

Centripetal acceleration of an object moving in a circle is given by $$a=\frac{v^2}{r}$$ where \(a\) is the acceleration, \(v\) is the linear speed of the object, and \(r\) is the radius of the circle.

Relative Centrifugal Force (RCF), for anyone like me who hadn’t heard this term, is simply the centripetal acceleration expressed as a multiple of \(g\) (the standard gravitational acceleration at Earth’s surface).

So quick rearrangement gives the minimum radius as $$r_\text{min}=\frac{v_\text{max}^2}{a_\text{max}}$$ which, plugging the appropriate numbers into wolfram alpha, comes out as about 2500 km, which is to say, about the radius of Mercury.

Plotted on a map using this tool, it looks like you could just about fit this loop inside Eurasia and still have it mostly on land (or with a bit of nudging, entirely inland):

Of course such a route requires the launch system to pass through a range of elevations from sea level to the Himalayas, and have high-performance electromagnets/charged rails/whatever along the entire 16,000km length. (For comparison, the LHC’s main beamline is about 85km long, and its radius is about 12km).

If the accelerator works on similar lines to the LHC, i.e. using a dipole electromagnetic to produce a constant magnetic field to accelerate a charged particle perpendicular to its motion, we can calculate how strong the magnetic field needs to be. the magnitude of the magnetic force on a moving particle in a perpendicular magnetic field is given by \(q v B\) where \(B\) is the magnetic field strength, \(q\) is the charge on the particle, and \(v\) is its speed. Equating this with the centripetal force needed, we get $$qB = \frac{ma}{v}=\frac{mv}{r}$$ so we can increase the acceleration by increasing the charge on the projectile, or increasing the magnetic field.

Let’s say for example we want to hurl a payload of the same mass that a Soyuz-2 rocket can lift to low earth orbit, which is about \(8000 \unit{kg}\) for a Soyuz 2.1b. We then need the product of the magnetic field and charge to be about \(31.4\unit{kg/s}\).

The maximum magnetic field strength used in the LHC is about \(7.7\unit{T}\). With this magnetic field strength, the charge on the payload needs to be about \(4\unit{C}\). This is a large, but not impossible charge (the Coulomb is a surprisingly big unit of charge). You would have to take care not to let that charge leak away.

A major problem is that the LHC beam is tiny, whereas we need to maintain a uniform \(7.7\unit{T}\)magnetic field across the entire charged section of the spaceship. Furthermore, the moving charge will radiate away energy as electromagnetic radiation, resulting in losses of energy. So an LHC style accelerator is probably not reasonable.

I’m not sure what the maths looks like for curved railguns or maglev trains on curved tracks, which are more realistic than a highly electrically charged spaceship.

Another issue is that your spaceship will lose a significant amount of energy punching through the atmosphere. I’m not sure exactly how much, and it of course depends on how steeply the track tilts before launch. And you will be launching at orbital speeds rather than slowly gaining speed as you pass through the atmosphere, so you will face the same sort of heat concerns as a re-entering spaceship, except even worse because the atmosphere will be even more dense at launch altitude.

To attain orbit, a space gun would not be enough - you would also need to carry a rocket to make a circularisation burn at the apoapsis of your orbit, or else your orbit will return to the launch point - but you will lose energy reentering the atmosphere, and therefore in practice land somewhere a long way short of the launch point.

For further considerations on space railguns (with a view to hitting the moon), I wrote a post some time ago, albeit concerning linear space railguns.

Despite all these problems, apparently there are at-least-somewhat-serious proposals to build a space gun, also known as a mass driver. Some proposals seem to involve a hybrid of a few launch systems: the space gun may involve both a circular and linear track and does not attain the full orbital velocity, but shaves off some of the delta-v needed by a conventional rocket.

Mass drivers seem to be particularly popular as a hypothetical way to launch on the moon: the required velocities are lower, and you don’t have a significant atmosphere to contend with. Here is one such proposal for a ring with a radius of about 100km and a maximum RCF of \(3g\).

Avatar
Avatar
reblogged
Avatar
canmom

Do planets close to the sun take more time to transit or less?

So yeah the transit of Mercury just happened, and I totally missed it. Oops. Not like I have a safe telescope projection rig anyway.

Nevertheless, this did make me wonder whether Mercury or Venus would take more time to transit.

Here’s some geometry, assuming a circular orbit for simplicity. We want to find the length of the orbital arc. \(x\) and \(y\) are angles because I couldn’t paste \(\theta\) and \(\phi\) into Inkscape for some reason. \(a\), \(r\) and \(l\) are known; the observer is stationary and coplanar with the circle.

So, peering at this diagram we say $$l \tan y = r$$ and also $$a \cos x = m$$ and also that $$m( \tan y + \tan x )=r$$ and the arc length is \(2ax\) (where \(x\) needs to be in radians of course).

OK, so let’s combine some things: $$a \cos x \left(\frac{r}{l}+\tan x\right)=r$$and that will allow a little rearrangement to $$\frac{ \cos x }{l} + \frac{\sin x}{r} = \frac{1}{a}$$

Well that’s like, probably impossible to solve analytically, but look we’re on a computer.

First of all here’s the angle \(x\):

So yeah \(x\) gets bigger as the orbit gets tight around the sun. I guess ultimately it’s going to go all the way up to \(\frac{\pi}{2}\) but Mathematica didn’t to show me that and it’s not important enough to fiddle around with Exclusions->None and all that.

OK let’s also check out the arc length

So the arc length is big when you’re in a really tight orbit round the sun, and otherwise just pretty much linearly decreases out towards the observer. cool

OK how much time does it take to cross this cone of vision? let’s grab Kepler. Kepler’s laws, which describe the motion of planets, say the square of the time it takes to go round the orbit (which we’ll call \(T\)) is proportional to the cube of the semi-major axis \(a\), which for a circular orbit is just the radius of the orbit. Neat. So that says $$T^2 \propto a^3$$

Of that time \(T\), a time \(t=\frac{Tx}{2\pi}\) is spent inside the transit visibility cone. That means $$t \propto x a^\frac{3}{2}$$ so let’s plot that thing.

So it looks like close to the sun is generally better, but there’s a local maximum some distance away from the sun. OK, what happens if we plot this in terms of more realistic relative sizes, i.e., set \(a\) to 1AU and \(r\) to the radius of the sun?

The hump is much bigger! And apparently for us Earth observers (or rather, a stationary observer at the orbit of Earth observing a coplanar circular orbit), the sweet spot is around 0.3AU.

Anyway that’s our answer for where it’s best to be if you want to spend a long time transiting the sun. Also, Mercury has a longer transit time than Venus. In reality it’s more complicated because orbits are elliptical and not coplanar, and the Earth moves during the transit too, so you only get transits sometimes and the time is a bit more complicated to calculate.

Avatar
reblogged
Avatar
canmom

Time Travel

In a no-longer-recent ask (gosh it’s been five months now), I answered a question with a fairly narrow scope: if you brought back a modern phone or laptop, what use would it be in The Past? Could you reproduce it?

But ultimately questions of ‘bringing stuff back’ depend rather a lot on the model of time travel you adopt. So this is a post about time travel models, paradoxes, possible connections between time travel fantasies and actual physics (albeit rather flimsy ones), etc.

Contents:

  • an attempt to list fictional portrayals of time travel, from the perspective of traveller and observers
  • discussion of models of causality that work without inconsistency, and what media portrays them
  • physics!
  • the ‘tachyonic antitelephone’, or why faster-than-light travel lets you send information back in time
  • a rough summary of time travel in general relativity
  • an attempt to make sense of where and when a time traveller ends up, and if time travel flings you into space
Avatar
reblogged
Avatar
canmom
Anonymous asked:

Assuming I have 16 goats and each acre of land can support 2-4 goats, any advice for making my dream of a lunar goat farm come true? (2-4 goats per acre is the suggested number of goats for poor quality soil, which I'm assuming Moon soil is when it comes to goat upkeep.)

this is just the sort of question I like to receive! I have a guess at who this might be (I have two anons who are big fans of goats).

Sadly, I think even poor quality soil may be overselling lunar regolith. I’m not fully sure what determines soil quality insofar as it pertains to goat farming.

There seem to be dozens of soil classification schemes that relate to all kinds of properties of the soil, from its strength and engineering properties to its material composition. We’re specifically interested in agricultural soil science but even that seems really complicated.

There is a portion of the lunar regolith referred to as lunar soil. Wikipedia has this to say of it:

The physical properties of lunar soil are primarily the result of mechanical disintegration of basaltic and anorthositic rock, caused by continuous meteoric impact and bombardment by interstellar charged atomic particles over billions of years. The process is largely one of mechanical weathering in which the particles are ground to finer and finer size over time.
This situation contrasts fundamentally to terrestrial soil formation, mediated by the presence of molecular oxygen (O2), humidity, atmospheric wind, and a robust array of contributing biological processes.
Some have argued that the term “soil” is not correct in reference to the Moon because on the Earth, soil is defined as having organic content, whereas the Moon has none. However, standard usage among lunar scientists is to ignore that distinction.

So it’s pretty unlike Earth soil. Here’s another comparison:

There are two profound differences in the chemistry of lunar regolith and soil from terrestrial materials.
The first is that the Moon is very dry. As a result, those minerals with water as part of their structure such as clay, mica, and amphiboles are totally absent from the Moon.
The second difference is that lunar regolith and crust are chemically reduced, rather than being significantly oxidized like the Earth’s crust. In the case of the regolith, this is due in part to the constant bombardment of the lunar surface with protons (i.e. hydrogen (H) nuclei) from the solar wind. One consequence is that iron on the Moon is found in the metallic 0 and +2 oxidation state, whereas on Earth iron is found primarily in the +2 and +3 oxidation state.

In terms of its practical consequences for creating goat pastures on the moon, I’m guessing lunar soil is going to be totally unsuitable for growing Earth plants. Crucially, it has almost no nitrogen at all.

Hopefully it’s possible to create fertilisers that can be mixed with lunar regolith to make something more suitable (taking care to mind the health risks of lunar dust) but if not, you’ll need to bring enough soil with you to cover your full 4-8 acres of land to a suitable depth. You will also need to bring significant amounts of water.

Wikipedia gives another long passage (sorry) on lunar agriculture:

Growing crops on the Moon faces many difficult challenges due to the long lunar night (354 hours), extreme variation in surface temperature, exposure to solar flares, nitrogen-poor soil, and lack of insects for pollination.
Due to the lack of any atmosphere on the Moon, plants would need to be grown in sealed chambers, though experiments have shown that plants can thrive at pressures much lower than those on Earth.[54]
The use of electric lighting to compensate for the 354-hour night might be difficult: a single acre of plants on Earth enjoys a peak 4 megawatts of sunlight power at noon. Experiments conducted by the Soviet space program in the 1970s suggest it is possible to grow conventional crops with the 354-hour light, 354-hour dark cycle.[55]
A variety of concepts for lunar agriculture have been proposed,[56] including the use of minimal artificial light to maintain plants during the night and the use of fast growing crops that might be started as seedlings with artificial light and be harvestable at the end of one Lunar day.[57]

OK, so, supposing you can create an area with an adequate soil quality. Let’s think about other considerations for goat farming on the moon.

There are a number of potential problems with constructing a lunar goat farm (or any moon colony). They’re discussed in a fair bit of detail in this wiki article. The main issues:

  • The moon is tidally locked to the Earth, meaning the lunar ‘day’ takes about a month. This leads to extreme temperature swings each month, and of course any attempt to grow crops would have them in darkness for half of each month. A potential solution to some of these problems is to place a lunar colony at one of the poles, since the moon’s rotation is very close to being perpendicular to the eclipltic.
  • Lacking a protective magnetic field and atmosphere, the moon’s surface is constantly bombarded with solar wind, UV radiation, and meteorite impacts of many sizes. For this reason, it’s often suggested to build a colony underground, perhaps in a lava tube.

All the same, this means we need to take care to keep the moon colony warm during the night and cool during the day, and it means we need to provide and store significant electrical power to keep the pastures illuminated for the half-a-month-long night.

It’s going to be very inefficient! Electrical power is inefficiently generated from sunlight or nuclear power, and is inefficiently re-radiated over the crops, is inefficiently used to produce carbohydrates in the grass by photosynthesis, is inefficiently digested by goats…

Living organisms are generally adapted to light and other environmental cues cycling over 24 hours (many biological processes display circadian rhythms). I expect we’d need to reproduce that in order to keep the goats happy and healthy. If our pastures are exposed to sunlight, we’d need to shield them; otherwise we’d need to turn the lights off at night.

Although the stereotypical sci-fi image is of a dome-shaped lunar colony, I think this wouldn’t be ideal. Domes, while good pressure vessels with no weak points, have a lot of air in them that’s too high off the ground to be useful (unless they’re very flat, in which case there’s a lot of inaccessible ground) that you still need to control the temperature of. Plus it would be difficult to cover up a dome with a shade to create artificial ‘night’. Instead, the above-ground habitat units would probably want to be as small as possible.

I feel inclined towards the underground moon colony concept instead. Our goat pastures would be arranged along a lava tube, or built in artifical caves.

Lunar lava tubes can be as wide as 500 metres before they collapse. Assuming we can cover about 80% of the circumference half of the lava tube with pasture before it gets too steep for the goats, the area of pastures of length \(L\) is \(0.9\times \pi \times 250 \mathrm{m} \times L\). For four acres of land, \(L\) comes out to only \(13\mathrm{m}\), which is very promising! The tube very likely wouldn’t be that big, and our pastures would extend further along it; all the same, this is a good sign.

More of an issue is the power of providing artificial light. Wikipedia’s above figure of peak 4 megawatts per acre would probably be an overestimate of what’s needed (also where on Earth is that calculated? Crops at the equator get much more energy than crops close to the poles!). Still, we’ll probably need a few megawatts of power to illuminate our crops. That would require a lot of solar panels, or else lifting a nuclear reactor to the moon.

While multi-megawatt reactors have not yet been launched into space, they have been thought about. In this old post I did on a drawing of a Soviet nuclear-powered airship, I found this report which discusses the possibility of 15MW reactors for use in space, which would more or less cover our goat farm. So nuclear power wouldn’t be out of the question.

So here’s my proposal for your lunar goat farm: we find a lunar lava tube. We place inside it some inflatable pressure vessels, containing enough soil to sustain a lunar goat farm (this will probably require quite a few lunar launches to place it all up there). Additionally we add areas to grow crops for ourselves? All water and waste is recycled back into the soil, but we may need to rebalance it with external inputs from time to time.

Keeping the ecosystem stable might prove difficult, however. So far, no-one has yet managed to create a stable enclosed ecosystem that can be mantained for a indefinite period.

We power it with a 15MW nuclear reactor weighing about 100 metric tonnes (this exceeds the trans lunar injection capacity of the Saturn V by a factor of two, but we might be able to reduce its mass by reducing shielding [since it will not be activated until it’s on the moon and we’re safely underground] and lifting it in parts), which will provide artificial light to the goats and the grass they’re eating.

The underground location provides protection from solar wind and other forms of radiation. The electric lights give us full control over the light in the enclosed environment, so we can keep our goats (and ourselves) in more or less Earth-like conditions.

Please do ask if you have any further queries about the lunar goat farm project.

Avatar
Avatar
reblogged
Avatar
canmom

What could you do with a portal gun or fenestrated pane/plane? Any really neat physics tricks that come to mind?

Avatar

Ooh, interesting question! I’m going to focus on Portal because my memories of fenestrated planes in Homestuck are pretty vague.

Actually I wonder a fair bit about how portals work! So my first inclination would be to test them out and find out.

For example, portals disappear if their substrate moves, but of course motion is frame-dependent. My inclination is to say acceleration is bad (since the distinction between accelerated and inertial frames makes sense when you don’t try to have gravity), especially since there’s a setpiece in Portal 2 with constant-speed moving portals which is the only exception to the ‘no moving portals’ rule.

But in general relativity, we are in a sense all being accelerated by the ground, which is why we are not following spacetime geodesics! Though really it would be better to say that there’s no fundamental distinction between ‘accelerated’ and ‘inertial’ frames in GR, because gravity and acceleration basically amount to the same thing.

Anyway, I’d try and investigate what causes the portals to disappear, for one thing. Perhaps that can’t be made to make sense, and it should be written off as a limitation of the game engine!

Next question: portals and fluids! Imagine you put one portal at the top of a column of water, another at the bottom. Across the portals, both pressure and gravitational potential are going to be discontinuous! (At least, initially - and the gravitational potential discontinuity won’t go anywhere.) What kind of flow does that imply?

arborine has shared with me a conversation some of her friends have had on this topic:

P:if you’ve got one portal on the ceiling, and the other on the floor below it, and you release a helium-filled balloon (the ordinary kid balloon size that’ll easily fit through a portal) up towards the ceiling portal… how will it behave?
A:actually, if the upper portal was significantly higher than the lower one, there actually would be a strong downward wind from the air pressure differential…
B: If they’re on the ceiling and floor of a normal sized room, there’d still be several kilograms of air between them, and the only thing stopping it from freefalling is the surrounding air via viscosity. There’ll be a downdraft, surrounded by a ring vortex. If the air ends up going down faster that the balloon can rise through it, the balloon will go down.

From that I’m thinking: with an incompressible fluid like water in the column, there will be no change in energy if water were to flow down into the portal. But if the fluid is compressible, like air, the denser air at the bottom portal would change the potential energy by being moved to the top of the column. Presumably the net effect would be to equalise density. Initially the discontinuous pressure and density would cause a shock front to propagate, but I don’t know what kind of stable configuration you’d be left with.

(Has anything like a portal ever been considered in fluids literature? Or, someone used to handling discontinuities in mechanics, what’s the standard way to treat them?)

[Portal 2 spoilers]A situation like that is portrayed in the game, at the end of Portal 2, where one portal is placed on the moon and the other on Earth. This causes air (and Wheatley and the Space Sphere) to be rapidly sucked out to the moon portal until the portals are closed. But then the moon is not connected to Earth by an air column so it’s not an entirely analogous situation.[end spoilers]

OK, then, another question is… if you can move portals, what happens if you put one connected portal into another portal? i.e. do this:

Because the geometry of that would get really weird, and I’m not sure if it could be consistent!

Based on how you can’t shoot the portal gun through a portal, I’m guessing that the portals would not pass through each other. Maybe you could do this, but it would get stuck on itself and you wouldn’t be able to push the portal in very far.

As for what you could do with portals… well, based on the tricks you use in the game, you could extract arbitrary amounts of energy out of gravity. e.g. put portals at each end of a coil of wire, drop a magnet in the coil. The falling magnet would produce an electric current in the coil and you can take as much energy as you want. (Of course, this is assuming a portal takes no energy to maintain!)

In Portal, objects that enter one portal emerge from the other as quickly as your computer can update their position. But then, the speed-of-light delay for information to travel from one portal to another required by special relativity wouldn’t show up on the scale of the game!

[Portal 2 ending spoilers] The scene where you shoot the moon presents more of a problem. In the developer commentary, they say that the time between the player firing a portal at the moon and the results of the moon portal being opened (air being sucked out the room, etc.) are based on the time it takes light to reach the moon.

However, to establish the pressure differential across the portals, information from the moon portal would have to travel back to the Earth portal, also at the speed of light. So (at least if I’m remembering what the devs said correctly) it should be twice the time before you see anything happen.[/spoilers]

Mind you, if there is a lightspeed delay between portals, that implies that widely separated portals are going to be displaying a situation some time behind the other portal. You could step into a portal seeing one thing on the other side, only to find the situation completely changed when you arrive after the lightspeed delay. Worse, the destination portal could disappear before you arrive! Where do you go then? Perhaps you are destroyed too.

So a question I’d want to investigate would be whether portals can be used to send objects faster than the speed of light. Because if they can, you can violate causality and indeed time travel physically as you please! Admittedly requiring a relativistic spaceship to do so on any helpful sort of scale.

But that reminds me of another question! Objects travelling through a portal maintain their momentum, except with the direction changed to match the new portal. Which necessarily violates global conservation of momentum. Could that be compensated for by conferring appropriate changes of momentum to the portals themselves? (This would of course break any ‘portals can’t be accelerated’ rule!)

If that is the case, you could use a portal as a propulsion system. Put a portal on the back of your spaceship, and a pressurised fluid behind the other portal, and you are constantly conferring momentum to your spaceship. Even if you can’t confer momentum this way, you can get around the Tsiolkovsky rocket equation by putting your fuel tanks on the other side of a portal. This makes space travel vastly easier, since the mass ratio of your spaceship is no longer exponential in the delta-v!

And of course, carry a portal on your rocket, and when you reach your destination you have a speed-oflight shortcut and there’s no need to send another rocket unless you need to send something too large to be carried in pieces through the portal. (though of course it would be much easier to deploy people and robots to make use of what’s on the planet!)

Of course, the 'shoot a portal at the moon’ scenario implies another means of space travel: if you can target your portal gun accurately enough, and there’s a suitable surface, you can place a portal on another celestial body and jump straight there, travelling at the speed of light with no observed travel time on your part.

Portals would of course allow a transportation network to be created on Earth. This would be an enormous change to the world’s economies, since transportation is almost no longer an issue for anything small enough to be transported through a portal. Instead of food being distributed by vast fleets of trucks and container ships, it could be passed through a portal directly from the point of manufacture. This makes me suspect portals would be very tightly controlled by people already powerful within capitalism.

And of course, portals would have wartime uses. Soldiers and bombs could be transported anywhere a portal could be placed along lines of sight, and supplies could be distributed with no possibility of interception. This is pretty scary to think about: imperialist countries with powerful armies would find it vastly cheaper to launch an invasion. If there was a nuclear war, it would be much easier for a portal gun to reach opposing cities than a nuclear warhead. Worse still if you can target a portal gun from orbit!

Presumably all airports would have Aperture Science Material Emancipation Grids to stop you setting up illicit portals. But that’s not very scalable! Portals would completely destroy borders, allowing much free-er undocumented travel.

Though per Portal 2 canon, you need to use moon dust to create aportalable surface, so you couldn’t place the portals just everywhere. (As people who’ve opened Half-Life 2’s maps in Portal know, as far as the game engine is concerned any concrete is just fine. But I don’t think that’s intended!) This would limit many of the uses of portals to people who could get moon dust. Of course, people would immediately use portals to set up lunar mining operations! And probably it wouldn’t take too long for people to figure out the necessary properties of moon dust to support portals.

GLaDOS wasn’t kidding when she talked about how valuable the portal gun was in the first game. Nothing would be the same with portals.

Anyway, unlimited free energy, much easier space travel, and complete reconstruction of the world’s economies is a pretty good start, I think. I would also of course use it to show up at sadjadewithcake’s house a lot for, um, things :p

Avatar
Avatar
reblogged
Avatar
dajo42

if the earth suddenly vanished what would happen to the moon

common consensus seems to be that it would launch along the tangent to its orbit at a linear trajectory towards some other gravitational force. so what i propose is that we make all the other planets vanish so their moons launch to us and we can have all of them

Goals…

Avatar
noctumsolis

I’m pretty sure that that’s dead wrong but I don’t know the equations.

No, I think that’s right; without the force of the Earth’s gravitation to pull the moon into an eliptical orbit, linear momentum would take over and the moon would just fly off at a tangent from its current position,

hi, certified doing-absurd-things-to-the-moon expert here

So, this is roughly right, the moon would indeed continue in whatever path its momentum at the time of the disappearance gives, and this would initially be tangential to its original orbit.

That said, however, the moon would still be orbiting the sun. It would not follow a linear trajectory, but an elliptical orbit with a focus at the sun.

How different would the moon’s new orbit be from the earth’s current orbit? The earth’s orbital speed relative to the sun is about 30 km/s, whereas the moon’s orbital speed relative to the Earth is ‘only’ about 1 km/s.

This means that regardless which direction it’s going around the Earth, the moon would barely modify its orbit around the sun. It would follow a very very similar orbit to the Earth - probably slightly more eccentric, but basically the same.

I was going to open up Universe Sandbox and illustrate this, but setting up a clear illustration of the Earth, Moon and Sun is surprisingly difficult, so that may need to wait.

Here is the orbit of the Earth (blue trail) and Moon (white trail):

You can see the Earth and Moon’s orbits are almost indistinguishable.

Now, let’s see what happens to the orbit of the Moon when we delete the Earth at the point shown:

This is after one orbit of the sun. The fading-out trail is the moon’s original orbit as part of the Earth-Moon system; you can see it’s kind of bumpy due to the Moon orbiting the Earth. The stronger white trail is the Moon’s new, post-Earth orbit. It’s definitely more eccentric (less circular, more elliptical) than the Moon’s original orbit, but still very close to the Moon’s orbit before Earth disappeared.

Avatar
reblogged

people who don’t believe in aliens confuse me so much like you really think in all of space, which is infinite, we are the only planet with life on it. ok….

(psst I don’t disagree with you about aliens but there is a finite amount of stuff in space. it’s very large but it’s finite. space is infinite in direction in the sense that if you travel for long enough, you’ll end up back where you started.)

i don’t know what any of this means i’m a one direction blog

Avatar
joshpeck

the one direction blog: Gemini, Aries, Sagittarius, Leo, Aquarius, Libra

the blog that knows about space: Cancer, Virgo, Pisces, Capricorn, Taurus, Scorpio

i hate this i’m a pisces

Avatar
theomenroom

Actually, if I understand space right (maybe one of my followers who studies physics can confirm?) the thing about ending up where you started if you go far enough is only true if the universe has elliptic curvature, and last I heard evidence suggested that it was flat overall (some parts are slightly elliptical, some parts are slightly hyperbolic, depending on the local balance of mass and dark energy or something and how that works out with general relativity, and overall it adds up to flat. of course literally all of that, except some of the geometry, is way over my head so)

We could have a positive curve universe, negative curve, or no curve. If I recall correctly, positive curve means it’s like a sphere ( if I’m wrong, positive means a saddle shape and negative means a sphere), and we are confident it’s not a sphere because if it were, we’d be able to see ourselves if we looked far enough in any direction?

That’s true but the visualizations don’t make a whole lot of sense if you don’t know the properties they’re trying to describe. Positive curvature is elliptic and negative curvature is hyperbolic, yes.

Let’s say you and a friend are holding hands in a space with some curvature. There’s some distance between you, and you’re walking in the same direction at the same speed, exactly perpendicular to your arms, and each of you is following your own line. In a flat (Euclidean) space, which is the most intuitive, you remain the same distance apart no matter how far you walk.

In an elliptic space, no matter which direction you’re walking (as long as you’re walking the same direction), you’ll end up closer together just by walking forward, until you bump into eachother. If you and your friend are geometric points with zero width, you will eventually collide at the “pole” corresponding to whatever line your arms were on initially. Also, if you keep your arms on the shortest distance between you, the angle between your path and your arms will shrink as your arms end up leading you. Once you reach the pole, continuing onward and turning back around may be mathematically equivalent, depending on the construction of the elliptic space you’re in (though if it is, that’s because going in any direction and its opposite is always equivalent. believe it or not that makes the math simpler). If it isn’t, once you do you will again put space between you, reach a maximum distance at the furthest possible point from where you started, and then approach another pole (and then go back to where you started). You can sort of see how this will work by drawing great circles on the surface of a sphere (circles which share a center with the sphere; meridians are great circles, parallels are not), which is why this is said to be spherical

Hyperbolic curvature, on the other hand, does the opposite. If you and a friend walk like this, your arms will trail behind you, and will also have to get longer the more you walk; just walking distance adds space between the two of you (and the further you go, the faster it adds space). Like in euclidean geometry, and unlike in elliptical, in hyperbolic geometry this just continues without end. There is more and more space the further you go.

People call this “saddle-shaped” because drawing lines on an (idealized) saddle and then measuring distance along the surface of the saddle has some similar properties to this, but embedding something that infinitely expands inside a euclidean space that doesn’t is hard.

so a few things to add:

in general relativity, the “curvature of space” is determined by the mass density there. however, on enormous scales, we can safely model the universe as homogeneous and isotropic, which has the result that the curvature is basically the same everywhere.

when we talk about whether the universe is infinite or finite, we are talking about the topology of the universe rather than its curvature, which is a local property.

if you look up the shape of the universe on Wikipedia, you will find it awash with potentially unfamiliar mathematical terminology such as ‘metric space’, ‘compact’, ‘bounded’, etc. I’ll try and write it more simply here.

  • the universe may be infinite, which is to say, there are points that are any distance apart, no matter how big
  • if it isn’t, it might have a boundary, or it might not. example of a 2D space that has a boundary: a disc. example of a 2D space that does not have a bondary: a sphere. Both of these have 3D equivalents.
  • we think the universe does not have a boundary, because we have no idea what it could possibly look like, how anything would behave there, etc.
  • so we’re left with a “compact metric space without boundary”. add the requirement that it’s “differentiable” and we can call it a “closed manifold”.
  • that, however, still leaves various possibilities - the universe may be “simply connected” like a sphere, or “multiply connected” like a torus.
  • this is kind of whether there are any holes in the space in a certain sense. basically if you draw any loop in space, can you shrink it down to a point? if you can, you’re in a simply connected space. if you can’t for some loops, you aren’t.
  • the topology of a space doesn’t tell you much about the curvature in any particular place!
  • for example, a regular polygon is topologically equivalent to a sphere. but the curvature of the space is 0 except at the points.
  • however, constant curvature of a space does place limits on the kind of topology the space as a whole can have.
  • the topology of the space gives it certain “topological invariants” and these can be related to the integral (basically like a sum) of the curvature at every point of the space
  • since we assumed the universe is homogeneous and isotropic, the curvature has to be the same everywhere. so whatever value we pick for the curvature will place limits on what topology the universe can have (e.g. sphere, torus, infinite)
  • please note: all of this is really hard to imagine because we, being creatures who perceive a 2D projection of a 3D world, are really not good at imagining a 3D surface embedded in four dimensions!
  • as noted in the previous post, if the curvature is positive, the universe has to be compact, whether or not it has a boundary. in fact it has to either be a 3-sphere or one of the “quotients” of the 3-sphere. anyway i guess this must mean that, you can head out in one direction and go ‘around’ the universe and end up back where you started. I do not recommend trying this.
  • if the universe is flat, it can be compact or infinite. if it’s infinite, you have Euclidean space. otherwise you can have things like 3D toruses or 3D klein bottles. (you can think of the 3D torus as being a universe that repeats endlessly in every direction.) There are, apparently, ten different possible topologies for a flat finite 3D universe.
  • if it’s negative, you’ve got hyperbolic geometry. wikipedia says “There are a great variety of hyperbolic 3-manifolds, and their classification is not completely understood.“ So that’s helpful. If you click on that link, it starts talking about knots and I stop being able to pretend I understand this.

so it’s pretty much as @theomenroom said but that’s a bit of extra detail.

now, as for the global topology of the universe of a whole? we actually don’t know!

the problem is that the universe is really fucking big. according to current cosmological models, the parts of the universe we can see now comprise a big sphere 93 billion light years across. (hold on isn’t that bigger than the 13.8 billion year age of the universe? yeah! because space is expanding, and the light we’re receiving was emitted when the universe was a lot smaller. also, relativity makes ‘now’ a kind of complicated thing. that distance is the comoving distance for the present time, if you’re worried.)

now, if the entire universe was smaller than the observable universe, we might see stuff repeating in the sky, in increasingly early instances. but if the universe is bigger than the observable universe, we can’t necessarily tell whether it’s finite or infinite! awkward.

what we do know is that the curvature of the universe is really damn close to flat. so we probably don’t have to worry too much about that spherical and hyperbolic stuff. but is it a 3-torus, or an infinite space, or what? we don’t know and maybe we never will!

cosmologists are trying to work it out though. they try to work out what effect certain shapes of universe would have on the things we can observe, such as the cosmic micorwave background radiation. so far: inconclusive.

another cool fact: in an infinitely big universe, there are, statistically, infinitely many copies of everything, though they’re arranged randomly. how far away is the nearest exact copy of the observable universe? we can estimate this! i’m trying to find a good source for the calculation, but an old scientific american article by cosmologist Max Tegmark says a copy of you within 10^(10^28)m, a 100 light year sphere (corresponding to a matching 100 years of history) 10^(10^92)m, and the entire observable universe should have a copy within 10^(10^118)m. for comparsion the observable universe is a mere 10^27m across.

These numbers are bullshit big - if i wrote then out in this post as 10000… we’d need 10^28 bytes to write even the smallest distance (the one to an exact copy of you), which is to say 10,000,000,000 times the estimated amount of data on the entire Internet. I tried putting 10^(10^28) into Wolfram Alpha to convert it to light years, and the calculation returned no results.

If you got in a spaceship and tried to travel to the clone universe at a speed arbitrarily close to the speed of light, before you got even the unimaginably tiniest fraction of a percentage of the way, all the stars in the universe would burn out, the particles would decay (depending whether protons decay), and only black holes would remain; the black holes would decay into particles through Hawking radiation, and eventually the universe would contain nothing but photons that almost never encounter each other. You’re still not noticeably closer. By the time you get halfway to where the clone universe used to be, a new Big Bang might happen apparently (idk how they worked that one out). Meanwhile, the expansion of the universe is moving the clone universe even further away, not that there would be anything left of it by the time you get there.

An infinite space contains points separated by any distance, and it will contain infinitely many copies of our observable universe - as well as infinitely many near-exact copies that differ by only a few particles, and so forth.

Anyway, we don’t know the overall topology of the universe - it might not be infinite, or big enough to contain any such silliness as an exact copy of the universe.

Avatar
reblogged
Avatar
canmom

A short(ish) post on Dyson spheres

“there is more tea in that sphere than the entire rest of the galaxy” - @rememberwhenyoutried

A Dyson sphere is a thought experiment about an engineering project: by encompassing a star in a sphere of solar collectors, all the energy it emits could be gathered. Dyson (who got the concept from earlier science fiction books, and wrote a paper about the possibility of detecting them) envisioned this as an inevitable creation of an ‘advanced’ civilisation with correspondingly enormous energy needs. For a star like our sun, that means trillions of times more than the current energy use of the Earth.

Dyson spheres show up in science fiction occasionally, usually as a solid shell of matter. (The post that inspired this one was a gifset of a solid Dyson sphere from Star Trek.)

That Wikipedia article is pretty substantial, and covers most of the things I might want to say about Dyson spheres. So all I want to do here is fill in some of the calculations the wiki article leaves out.

Statites (non-orbital satellites)

One of the proposed forms of a Dyson sphere is a swarm of ‘statites’ - enormous solar sail collectors held in place against gravity by radiation pressure. Wikipedia offers a value of the area density that would be necessary for a stable statite.

I have two questions about this: how was this value calculated, and is a statite stable against small peturbations to its orbit?

Calculating the value is pretty straightforward actually - I did it in physics in like second year?

So: solar radiation pressure. Photons carry momentum, depending on their frequency. To be precise, a photon carries momentum $$p=\hbar k=\frac{2\pi \hbar f}{c} =\frac{E}{c}$$ (where \(f\) is the frequency, \(k\) is the wavenumber, \(E\) is the energy of the photon).

Note that the momentum of the photon depends only on its energy. So we can just take the total energy flux on a surface, divide it by the speed of light, and get the force (rate of momentum change over time) per unit area. Sweet!

If a photon hits an object and is absorbed, to balance momentum, the object must gain an equivalent amount of momentum. If it’s reflected back in the opposite direction, the object has to gain twice the momentum of the photon. Which means the optimal situation for a solar sail is a perfect reflector that sends the incident photons exactly where they came from.

OK, so, now, how much force would the solar sail on our statite receive? Suppose the sun radiates a total power \(L\). At a distance \(r\) from the centre of the sun, this is spread over a sphere of area \(4\pi r^2\). So the power per unit area received at this distance from the sun is just $$P=\frac{L}{4\pi r^2}$$ and the force per unit area imparted on the surface, assuming perfect reflection, is $$F_\text{light}=\frac{2P}{c}=\frac{L}{2\pi c r^2}$$

Now, the solar sail is also attracted by gravity, which also follows an inverse square law! The force per unit area from gravity is merely $$F_\text{grav}=\frac{GM\sigma}{r^2}$$where \(\sigma\) is the surface mass density (i.e. mass per unit area).

If our statite is indeed static, the mass per unit area must equal the force of light per unit area, or else it would accelerate. So we can set \(F_\text{light}=F_\text{grav}\) which means $$\frac{L}{2\pi c r^2}=\frac{GM\sigma}{r^2}$$and after a little rearrangement we get that mass density we were looking for, $$\sigma = \frac{L}{2\pi c GM}$$

Plug in the solar luminosity and mass, and you get the mass per unit area of your statite as \(1.53\mathrm{gm^{-2}}\) which is really very extremely light!

A check: this is twice the Wikipedia quote, which cites this Dyson sphere FAQ which makes the same calculation we did, but reports the force of light as half what we had. Although they’re claiming this represents a perfectly reflecting sail, it looks more like they’re giving the result for a perfectly absorbing sail?

I checked my formulae against the numbers in the solar sail article and I believe the calculation above is correct, or at the very least, mistaken in the same way as whoever wrote the solar sail article.

Note also the solar sail would have to be lighter, per unit area, than that value, to account for rigging, electronics, etc. And this is already absurdly light: Wikipedia’s speculation is going as far as suggesting ‘single sheet of graphene’ as a solution.

Is it stable? Actually this is easily answered: the force balance above does not depend on the distance to the sun, so if it moved from its position slightly, it would not start drifting away. However, if the solar sail rippled or rotated slightly from directly facing the sun, it would fall towards the sun slowly; if the solar sail was slightly too big, it would start to blow away from the sun. The statite would have to monitor its position and adjust the sails constantly - but there’s no reason it couldn’t do that.

Solid dyson sphere

So the first thing to repeat is that as a result of the shell theorem, a solid spherical dyson sphere would not interact gravitationally with the sun, and could therefore drift until it hit the sun.

How strong would it have to be? This is discussed in the Dyson sphere FAQ above: they suggest considering the dyson sphere as two hemispheres joined at a seam. Each hemisphere, attracted by the sun, would press against the other: the dyson sphere must be strong enough to resist the pressure from both sides.

How strongly is a small piece of the sphere, of mass \(\rho t \dif A\), attracted towards the other hemisphere? Here I’m saying \(\rho\) is the density of the Dyson sphere material, \(t\) is the thickness of the sphere, \(\dif A=r^2 \sin \theta \dif \phi \dif \theta\) is an element of area with the spherical polar coordinates shown in this picture:

The force towards the sun is \begin{align}\dif g &= GM \rho t \dif A r^{-2} \\ &= GM \rho t \sin \theta \dif \phi \dif theta\end{align}which is, interestingly, independent of the size of the sphere! The net force in the direction of the other hemisphere is \begin{align}\dif F &= \cos \theta \dif g \\ &= GM \rho t \sin \theta \cos \theta \dif \phi \dif \theta \\ &= \frac{GM \rho t}{2} \sin 2\theta \dif \phi \dif \theta\end{align}

We now need to integrate this over the entire hemisphere to get the force at the join from one hemisphere: \begin{align}F&=\frac{GM \rho t}{2} \int\limits_{\theta=0}^{\pi/2} \int\limits_{\phi=0}^{2\pi}  \sin 2\theta \dif \phi \dif \theta \\ &= GM \pi \rho t \left[-\frac{1}{2}\cos 2 \theta\right]_0^{\pi/2} \\ &= \frac{GM \pi \rho t}{2}\end{align}so in fact all we did was insert a factor of pi, huh. Since both hemispheres are pressing with this force, the total compressive force at the join is \(2F=GM \pi \rho t\).

The area of the ‘seam’ between the two hemispheres is, assuming the sphere is thin compared to its size, just \(2 \pi r t\). So the pressure on the seam is $$P=\frac{GM\rho}{2r}$$ regardless of the thickness of the sphere (as long as it’s not so thick that the thickness is comparable to its radius!) We can compare this with the compressive strength of the material.

If the sphere is made of steel, for example, and the sphere is \(1 \mathrm{AU}\) in size (which seems to be the standard example, though it sounds unnecessariy large to me - but also note that the larger the sphere, the less strong the material must be), we find the pressure is \(3.5\mathrm{TPa}\) which is helpfully declared to be nine times the pressure at the centre of the Earth, and seven times the highest pressure sustained in a laboratory. It is, therefore, quite a lot more than the compressive strength of steel, which varies depending on how the steel is made and what materials are used, but might be in the range of hundreds of megapascals and therefore tens of thousands of times smaller what would be needed to hold up a Dyson sphere.

There are two options for making the strength requirement of your solid Dyson sphere more reasonable: make the sphere bigger, and make it less dense.

If you used one of the lightest materials known to humanity, carbon nanofoam, the compressive strength requirement goes down to \(0.28 \mathrm{TPa}\) which is still absurdly outside the realm of possibility.

On the other hand, if you doubled the size of the sphere, you’d halve the strength requirement, but quadruple the amount of material you need.

Worse, if something causes your sphere to buckle towards the sun, gravity will tend to stretch out the distortion, potentially tearing a big hole. Big problem!

This is one of the major reasons a solid dyson sphere is considered one of the least possible kinds of dyson sphere, out of a range of things that are already pretty absurd.

How hot is a Dyson sphere?

Ultimately, whatever solar energy you gather from your Dyson sphere will eventually become unusable heat that you have to radiate away into space to avoid heating up. This is what Dyson was looking for in his paper: a Dyson sphere would radiate a different spectrum of radiation to a bare star, so you could potentially recognise one with a telescope. (People have tried, and not found any evidence of one.)

The radiative temperature of a Dyson sphere in equilibrium depends on its size. The usual rule for emission of something a bit like a black body is that the power emitted is $$L=4 \pi r^2\sigma \epsilon T^4$$where this time \(\sigma\) is the Stefan-Boltzmann constant, not the mass density, \(\epsilon\) is the emissivity, a number between \(0\) and \(1\) (with \(1\) being a black body) that tells you how well the material emits light, \(T\) is the temperature, and \(r\) is the radius of the spherical body.

The energy coming in to the Dyson sphere (from the star inside) is constant. So you have $$T\propto \frac{1}{\sqrt{r}}$$i.e. the temperature decreases as the inverse square root of the size of the sphere. If we put all the other constants in we get $$T=\sqrt[4]{\frac{L}{4\pi r^2 \sigma \epsilon}}$$and for a \(1 \mathrm{AU}\) Dyson sphere that is a perfect emitter, we find the temperature is about \(120^\circ \mathrm{C}\).

I want to make some metaphor about how like, if you poured water on the surface of the Dyson sphere, it would boil, but you’re in space so that would happen anyway. So instead um, if you had like, a cabin with 1 atmosphere pressure inside, and you had some water in there, it would boil.

If you want to reduce the temperature, make your Dyson sphere bigger, or make some holes in it to allow some of the heat of the sun to escape directly.

I think those are all the questions I had about Dyson spheres answered. For other questions, such as how much material it would take, have a look at the links above, they might cover you! (You are essentially talking about dismantling the entire solar system.) Or ask me. I like talking about stuff.

Avatar
reblogged
Avatar
canmom

What are the planets up to?

Astrologically-minded people have been talking a bit about Mercury going into ‘retrograde’ recently.

I’m sure if you look you can pretty easily find an explanation of what that means in astronomical terms (I won’t try to speak to the astrological significance of it in whatever tradition you might subscribe to), but regardless I thought it would be nice to do a big long post about how the planets move around in the sky, how we know in advance what they’re going to do, etc.

This won’t be a post about the historical development of these ideas. I already did one of these. And I’m gonna include some maths, but also lots of words, so hopefully if you’d rather, you can just ignore all the equations!

Perspective view of the orbits of the main bodies in the solar system - image by Andrew Z. Colvin on Wikimedia commons

OK, so what’s going on up there then?

So in general when you want to work out the answer to some question like “what are the planets up to”, physicists want to be able to start with the where the planets are at one point in time (this is called the “initial state”) and get a description which tells them exactly where the planets are at every particular time before and after.

That’s the ideal.

The general problem of a bunch of objects (three or more) moving around and interacting through gravity is “chaotic”. That means that our prediction of where the objects go next can change a huge amount when the initial conditions change. Because of this, you can’t in general get a nice straightforward set of equations that tell you what‘s going to happen.

So you have to make some approximations in your description of the system, and use a computer to predict approximately what will happen. If you’re lucky, your approximations won’t lose anything important.

The lucky thing about the solar system is that the sun is much much much bigger than everything else. So while each planet is attracted not just to the sun but every other planet in the solar system, the effect of the sun is vastly bigger than that of every other planet.

A very good approximation, then, starts by assuming the sun is the only thing that matters, and just ignoring all the other planets. That’s really the only way you can go if you don’t have a computer simulation handy.

Kepler’s model, which is what happens if the planets all ignore each other

(hamiltonianflow has done a very good series on this subject, which you can read here. So if you want a bit more mathematical detail than I go into, try her series!)

Here’s an example of the kind of shape called an ellipse.

When you only have two bodies involved - for example, a planet and the sun - each body orbits along an ellipse. (Or some other kind of conic section - but if it’s not orbiting along an ellipse, the planet will fly off somewhere very far away, so let’s not worry about that.)

The point marked \(F_1\) is called a focus (plural foci) of the ellipse. \(F_2\) is the other focus; one definition of an ellipse is that if you take any point along the ellipse and add up the distances to the two foci, the total will be the same.

What’s important is that, for the ellipses that the objects two objects are moving along, each has one focus at a point called centre of mass.

The centre of mass is a point that’s nearer the more massive (heavier) of the two bodies (and bang in the middle if they’re the same mass). Now, the sun’s mass is \(1.99\times10^{30}\thinspace\mathrm{kg}\). The mass of the heaviest planet, Jupiter, is \(1.90\times10^{27}\thinspace\mathrm{kg}\). That means that the sun is a thousand times heavier than Jupiter; the difference between the sun and the other planets is even bigger. So the centre of mass is pretty much right in the centre of the sun, for every planet.

This means you can pretty accurately say each planet orbits along an ellipse, with its focus at the centre of the sun. (That’s called Kepler’s First Law, because Kepler came up with this model first.)

An ellipse tells you where the planet might be, but we want to know where the planet is at a particular time. Kepler had a complicated description that comes in terms of ‘areas swept out’ by a line between the planet and the sun. Nowadays, we treat it with various equations, though finding a simple equation to get the orbital state vectors as a function of time is surprisingly difficult!

But the important detail is basically that planets travel faster when they’re near the sun, and slower when they’re far away from the sun.

That particular relation - speed vs. distance from the sun - is expressed by quite a simple equation called the ‘vis-viva equation’,$$v^2=GM_\odot \left(\frac{2}{r} - \frac{1}{a}\right)$$In this equation, \(v\) is the speed of the planet, \(r\) is its distance from the sun, \(a\) is the semi-major axis which is a measure of how big the orbit is, \(M\) is the mass of the sun, and \(G\) is a constant that tells you how strong gravity is.

The third thing Kepler said is that the total time \(T\) that a planet takes to complete an orbit of the sun - its ‘year’ in other words - grows in a particular way depending on how far away the planet is orbiting:$$T^2=4\pi^2\frac{a^3}{GM}$$or the square of the period (year) is proportional to the cube of the semimajor axis.

So the length of the planet’s year depends how far away the planet orbits from the sun.

Those three rules are enough information to tell you exactly where the planets are going to be at any point in the future, assuming the sun is the only thing that matters.

So what about our planets specifically

Our sun formed from a giant disk of gas and dust that slowly shrank under its own gravity around a dense core. Because of a principle called ‘conservation of angular momentum’, as the disk shrank, the tiny motions of the gas within the disk are increased to form a large, flat, rotating disk.

A lot of that disk is picked up by the growing sun in the centre, but the leftovers form small lumps which grow into bigger lumps as they collide with each other. These eventually form the planets and the asteroids.

The result of this process is that the planets all rotate in the same direction around the sun, and they all orbit in almost exactly the same plane. This plane is called the ecliptic.

The orbits of the planets in our solar system are not very eccentric at all either. The eccentricity of an orbit is basically a measure of where the ellipse falls on a continuum from ‘circle’ to ‘straight line’, so a more eccentric orbit is one that’s more stretched out. So in other words, the orbits of the planets are, despite being ellipses, very near circular. This is part of why models of the solar system featuring circular orbits were very effective.

As for why the eccentricities are so low, there are a few possible answers - interactions with the other planets, interactions with dust in the protoplanetary disc, tidal interactions with the sun - but it’s still an area of active research.

imageimage

A gif showing the above-described orbits from here, via Wikipedia

The sun is not the only thing that matters

You can get away with ignoring the influence of the other planets over a few centuries, but eventually the effects of their small gravitational pulls starts to add up. Which leads to surprisingly large consequences for things like the climate of the Earth!

Because the sun is still the most important thing in the picture by far, we usually look at these orbital peturbations as modifications to the picture we just described.

These can come from a few things:

  • the other planets (the big one!)
  • things not being spherically symmetrical
  • things getting caught up in atmospheres
  • General Relativity

Because a lot of the causes of these peturbations (i.e. other planets coming near) are periodic - they happen regularly with the same time between them - the peturbations are also periodic. (The planet Neptune was discovered as a results of the small periodic peturbations it was causing to the orbit of the planet Uranus).

For planets’ moons - particularly the Earth’s Moon - things get flipped round: the Earth is the main thing in the picture, but the sun causes the biggest peturbations and changes the Moon’s orbit quite significantly over time.

It is also the case, in theory, that these peturbations can add up and throw a massive spanner in the nice neat orbits we’ve been talking about so far, leading to planets colliding with each other.

We can ask if, over millions and millions of years, the behaviour of planets in the solar system is stable (i.e. the only changes are the small peturbations, but everything remains much as described above). The answer is: we know pretty much exactly everything that’s going to happen over the next few million years, and we don’t have to worry about colliding with a planet.

For the next few billions of years, things are probably going to be ok, but if we give it five billion years, there’s a tiny chance that Mercury’s orbit will go wildly unstable, either plunging it into the sun, colliding it with Venus, or disturbing Jupiter in a way that fucks up the orbits of all the inner planets so we could collide with just about any of them. Fun. In any case, after that 5 billion year point, the sun will run out of hydrogen to effectively fuse in its core, and grow into a red giant star, swallowing up the innermost planets and destroying any remaining life on Earth.

(For comparison: life on Earth has existed for about four billion years, and we humans have been around for 50k-100k years, which is to say, 0.0025% of that. It is near certain that all humans will be billions of years dead by the time Mercury and the sun get around to doing any of those things. And if by some impossibility we have not, I would expect us to exist on other planets apart from Earth. So don’t worry too much :3)

One thing on that list, general relativity, only gets really important when you get near the sun. The very brief explanation is that Newtonian gravity is actually only an approximation itself to a much more mathematically complicated theory. On the scale of the solar system, that mostly doesn’t matter, but it does have a small but noticeable effect on the orbit of Mercury! That effect is one of the lines of evidence that formed some of the earliest experimental tests of the theory, so I thought it was worth a mention.

As for the climate thing - well, the peturbations to the Earth’s orbit and the way in which its spinning are generally periodic, repeating over thousands of years. Examining long-term historical climate data revealed these incredibly slow changes (exaggerated by feedback effects) lead to similar periodic changes called Milankovitch cycles in the Earth’s average temperature. Probably. This science is perhaps a bit less settled than some of the rest of this post.

What do we see of all this from the ground?

Since most of the people reading this post are probably on the surface of the Earth (any astronauts and/or extraterrestrial lifeforms reading, please say hi!), if we want to observe the planets, we look up at the night sky for little bright dots that look very much like stars. (If we are lucky enough to have a good telescope, we can resolve these dots into flat disks with some surface variation.)

Most of the things in the sky, as watched by human eyes from Earth, don’t do much, day to day and month to month. They sit in a fixed pattern in relation to the other stars. Now we have telescopes, we can notice that isn’t quite true - some of the more nearby stars move a bit over the course of the year due to ‘stellar parallax’ which lets us work out how far away they are, and some stars are moving through space quickly enough to notice in a telescope (particularly the stars orbiting the black hole Sagittarius A* at the centre of our galaxy!).

The planets are not like these ‘fixed stars’. They don’t stay in a set position compared to the fixed stars. The English word “planet” is derived from a Greek word meaning “wanderer” for this reason.

When something is odd and moving about, a natural thing to do for people across the world was to keep track of where it is each night. And if you do this, you can see the planets slowly make their way from place to place in the sky, reflecting their motion in space. Planets close to the sun, like Mercury, generally move much faster than planets far away from the sun, like Saturn. (Uranus and Neptune are harder to see without a telescope.)

We can imagine the fixed night sky as being an enormous sphere centred on the Earth, with all the “fixed” stars as little dots attached to it. This is helpful for describing how the planets move in the night sky. (The sphere is basically a map of the angular direction of each star from the centre of the Earth.)

So in this picture, the Earth is sitting in the middle of the frame, with the celestial sphere centered on it. I’ve drawn two planets, and two lines through those planets to the celestial sphere, to show how the positions of planets in the celestial sphere is determined by the relative positions of these planets.

How do the planets move? Here’s a picture where I’ve drawn this diagram for two different days, one over the top of the other…

The positions of the planets on the first day and its celestial sphere is faded out slightly. I’ve also drawn the day-1 positions of the planets on the day-2 celestial sphere, again faded out. I’ve made Venus move further than Earth move further than Jupiter, but I’m not doing this at all exactly.

Now all three planets have moved around the sun, both Jupiter and Venus have moved to slightly different places on the celestial sphere.

That’s the general principle of how planets appear to ‘wander’. You can see it’s pretty complicated.

Now, a very large part of the motion, especially for the inner planets, comes from the Earth’s own orbit. Because of the Earth’s orbit, these planets appear to travel all the way round the night sky over the course of a year, even though their movement in space is much smaller than that of the Earth. This is why it made a lot of sense to people, at first, to say the planets travelled round the Earth.

Because the planets’ orbits are largely confined to a plane (the Ecliptic mentioned above), they all pass through roughly the same areas of the night sky. As a result of the Earth’s motion, so does the sun. (Based on this property, the predominant Western astrological system divides the night sky into 12 equal-sized regions, into which the planets pass over the course of a year. Patterns of stars in each of these regions (as seen from Earth) give each one its symbolism).

And retrograde motion?

One of the interesting aspects of living in a heliocentric solar system like this is that, on occasion, the planets seem to temporarily change the direction of their motion relative to the fixed stars in the night sky. They can follow a pattern such as this over the fixed stars:

(this squiggle shows the motion of Mars in 2009-2010, via Wikimedia commons)

This presented a big headache to people trying to work out a geocentric model of the solar system (one in which everything rotated around the Earth) according to Aristotle’s principle . In a heliocentric solar system, such as Kepler’s one described above, there is a straightforward explanation, though the situation looks slightly different for planets orbiting inside the Earth and planets orbiting inside.

For outer planets such as Jupiter, the situation is this: Jupiter is sedately carrying on its orbit, and the Earth zooms past on the inside.

This image (by Rursus on Wikimedia commons) shows the positions of two planets, the Earth (the ‘T’ planet) and another planet (’P’) as well as the other planet’s position on the celestial sphere, at each of five points in time over the coures of half an Earth year. Unlike my diagrams, Rursus has centred the celestial sphere on the sun, and not the Earth. The difference stops being important as you make the celestial sphere bigger, and it does make it a lot easier to show everything on one diagram.

Anyway, as the inner planet moves up towards the outer planet, they are moving at quite different angles, so the outer planet appears to move a large distance on the celestial sphere. As the inner planet catches up, being faster, the outer planet appears to move ‘backwards’, until eventually the inner planet is moving more or less directly away from the outer planet and it appears to move forwards again.

To show that this still happens if we use a properly Earth-centered celestial sphere, I did a series of images :)

…which, however rough they are, hopefully show the general principle.

For a very slowly moving outer planet, this happens nearly every Earth year. For a more quickly moving planet such as Mars, it will happen slightly less than once a year.

For the planets closer to the sun than Earth (Mercury and Venus), you can essentially draw the reverse of this diagram, fixing the outer planet in the frame as your ‘Earth’ and drawing lines through the inner planet. That said, to see retrograde motion when I tried this, I had to tune the speeds a little bit - but with the rigt speed, it can be quite pronounced. (I’m not going to post the whole series, but this image was constructed the same way as the last set!) The red squiggle is a rough trace of the positions visited in order, in order to make the retrograde motion clearer.

Anything else I should know about those planets?

While I was talking, Neptune stole your wallet. Sorry.

More seriously, there’s a lot more mathematical detail to be had if you want to learn more about the planets. I represented stuff as 2D circles and the celestial sphere as a circle, but of course we live in 3D space (3+1D spacetime if you want to get all relativistic on me), and the sky forms a 2D plane, so you have to invoke a lot more parameters to describe the motion of the planets of the sky in all its precise grisly detail. To fully describe an elliptical orbit, you need six numbers. To precisely state the position of objects in the sky (in terms of “point your telescope here”), there are many coordinate systems. Like, really a lot.

Also I guess… I have no wish to be the ‘no fun allowed’ person, but if you’re genuinely worried about Mercury going into retrograde (I can never tell quite how serious people are about that, but… I know what it’s like to have anxiety :/), astronomically there’s nothing more going on at present than an interesting visual effect that becomes apparent if you very carefully keep track of the position of Mercury in the sky over a long period of time. There’s no special reason why things are more likely to go wrong while Mercury and the Earth are in these particular parts of their orbits.

Avatar
reblogged
Avatar
canmom

assuming a starship never has to travel through an atmosphere, and never has to worry about collisions with eg. asteroids or attackers' weapons (maybe some sort of forcefield, or a perfect interceptor weapon system), are there any compelling reasons for starships to be any particular shape? (let's also assume the starship can travel at relativistic speeds)

Avatar

Aha! Excellent question! I love talking about spaceships.

So most of what I know comes off Atomic Rockets, which has far too much love of old whitedude science fiction novels, but also a lot of useful information, and more recently, lots of conversations with arborine, who has thought/knows a great deal about space habitats and is generally wonderful.

There are a few design considerations that constrain what shape your spaceship should have…

  1. it needs to bear the stresses of acceleration without breaking
  2. it needs to dissipate the heat generated by the crew, onboard machinery and direct sunlight into the highly insulating vacuum of space
  3. it needs to be able to manouvre correctly and, for example, not start spinning uncontrollably when the engine does a burn
  4. its rotation needs to be long-term stable in flight
  5. it needs to be able to withstand impacts with interplanetary/interstellar debris, which although consisting only of rarified gas and small particles, may (depending on the mission) may be approaching at extremely high, even relativistic speeds
  6. it needs to protect equipment and crew from space radiation (from stars or cosmic rays)
  7. it needs to protect the crew from any radiation produced by the ship’s engines, or its power source (a big issue if you use something scary like nuclear power or antimatter annihilation!)
  8. if humans are on board it needs to have pressurised compartments for them to live in, and enough space that they will be comfortable for the duration of the voyage
  9. it needs to minimise the extent of the damage if something goes wrong, e.g. part of the ship is depressurised
  10. it needs to survive for the entire length of the voyage, which probably means it needs to be self-sustaining
  11. it needs to carry enough propellant reaction mass to give it the \(\Delta v\) for whatever it’s out there to do
  12. it needs to be as light as possible, because every unit of mass of spaceship also adds a lot of units of reaction mass, depending on the mission \(\Delta v\)
  13. if you want the crew to experience artificial ‘gravity’, it either needs to be capable of accelerating near \(g\) for nearly the entire mission, or have sections of the spaceship capable of rotation to create a centrifugal force near \(g\)

I’m sure there are a lot more of them.

So while there’s no reason for your ship to be streamlined, there are a bunch of constraints!

1. make it strong, and maybe put the engine at the front to save mass

The first one depends a lot on where you position your rockets. Most designs place the propulsion at the back of the ship, pushing the ship forwards.

If you put the propulsion system at the back of the ship, you need your ship to be sturdy against compressive strength, which adds to the mass of the mission - compressive strength usually requires massive beams and so on, and it’s hard to get around that. When you’re accelerating, it will feel as if gravity is pushing down on everything on board towards the back of the ship, and like a building, your ship has to be strong enough to stand up. Otherwise, your engine will punch straight through your ship, or crush it, or tear off, or something similarly bad.

A tiny number of designs, like the Valkyrie, and in fact some of the earliest rockets ever built, have the propulsion at the front of the ship, towing the rest of the ship behind it.

If you do that, your ship needs tensile strength to not snap against the propulsion. You generally don’t need as much mass to have tensile strength as compressive strength - but putting the engine at the front does mean your crew are directly in the path of the engine exhaust! (The radiation problem is a general one for nuclear and antimatter engines, but it’s particularly severe when you’re right in the engine beam…)

The Valkyrie gets around this by having a shadow shield between the engine and the crew compartment, which absorbs radiation from the engine and casts a shadow over the crew compartment.

Another option is to have your engines not point directly backwards, but have two or more engines angled slightly out to the sides in opposite directions - this is apparently what the ship in the film Avatar does (ship’s presumably a still from the film, annotations from Atomic Rockets).

Putting the engines at the front on a flexible tether does make it much harder to change direction, though.

2. make sure it has large flat surfaces, or sprays of recoverable droplets, to act as radiators

Although space is sometimes described as ‘cold’, especially in movies, that’s kind of misleading. Certainly, if not in direct sunlight, something in space without a heat source will eventually cool down until it’s incredibly cold… but for the same reason that we use vacuum flasks to keep tea or whatever warm, it takes a very long time.

If you’ve got something like a spaceship, which is generating a lot of heat, you need to do something to dissipate that heat. The only mechanism that heat can be lost in space, with no atmosphere to carry it away, is electromagnetic radiation. To lose heat through radiation, you need a large surface area (i.e. big flat surfaces called ‘radiators’), and to carry the heat from the rest of your ship to the radiators. You heat up the radiators and they radiate away your heat into the vacuum.

The Space Shuttle would apparently open its cargo bay doors in space to help it radiate away heat (labels once again from Atomic Rockets).

Your radiators should not have their surfaces facing each other, or else they will heat each other up, and won’t radiate heat as effectively.

Another trick to cool your ship down is to use a ‘Liquid Droplet Radiator’ (picture source - apparently the ‘ICAN-II’ design from Penn State University, no idea what that stands for). This involves using your waste heat to heat up a spray of hot liquid droplets, which travel through space, steadily cooling down. As your ship is accelerating, the droplets ‘fall’ towards the back/front of the ship (depending which way you go), and can be collected, heated up and sprayed back in the opposite direction, maintaining continuous circulation.

Atomic Rockets has some really weird-looking designs, including discs, triangles, and even one that looks like a spiral (using magnetic fields).

Have a look at Atomic Rockets - there are lots of radiator designs I haven’t covered, involving bubbles, nanotube filaments, all sorts of stuff.

3. make sure the thrust vector goes through the centre of mass

If it doesn’t - if your engine is off-centre - when you do an engine burn, it will also have a nonzero moment (torque). This means your ship will start to rotate around its centre of mass when you do an engine burn. Sometimes, this is what you want, but usually only by a small amount! And typically a large engine burn would set you spinning wildly. So you will probably have separate small thrusters for turning.

(This gets even more complicated if your ship has flexible elements!)

4. make sure that, if your ship is spinning, it is spinning around its major principal axis

Every rigid body has a thing called its moment of inertia tensor that can be calculated from its mass distribution. Its eigenvalues are called the principal moments of inertia, and its eigenvectors are called the principal axes of the body.  When the engine is not burning, the ship is undergoing free precession.

I remember a series of arguments about free precession in our classical mechanics lectures that say, since a not-quite-rigid body slowly loses energy to internal stresses but the angular momentum is conserved, it precesses (the angular velocity vector moves with respect to the body coordinates, if that means anything to you) so as to end up spinning around the principal axis with the largest moment of inertia.

If your plan is that your spaceship should spin around a different axis, you should be worried, because it will slowly precess to spin around the major axis.

At the time the USA launched their first ever satelite, Explorer 1, in 1958, this was not known. The pen-shaped (shh) satellite was set spinning around its minor axis, the long axis through the middle of the spaceship. To the scientists’ surprise, the axis of rotation changed in flight, until the satellite was flipping end over end. This led to the first development of Euler rotation (the kind of thing we’re talking about) in over 200 years. But we’ve done that now, so, make sure your spaceship is spinning around its major axis if it spins!

5. if you’re going too fast, the ship needs to be safe from relativistic dust

A tiny grain of dust, when raised to relativistic speeds, can have the kinetic energy of a bullet or bomb. And, if your ship is travelling through space at relativistic speeds, every grain of dust in space will be approaching your ship at those speeds! This is bad.

One solution is to put a shield at the front of your spaceship, which will be punched up by relativistic dust, but will disperse most of its energy before it smashes anything important.

I’m going to talk about the Valkyrie again. I am maybe a bit too into this design, but their solution is really cool.

The Valkyrie sprays fluid droplets ahead of the ship. Any incoming particles can detonate harmlessly among the droplets, without touching the engine. Because the ship is constantly accelerating, the fluid droplets will ‘fall’ back down and land relatively slowly on the front of the ship, where they can be recycled. This is also used for cooling - another ‘liquid drop radiator’.

During the deceleration phase of the mission, this doesn’t work - the droplets fly off ahead of the ship instead of being recaptured. The Valkyrie’s solution is to grind its spent fuel tanks into fine dust and release them ahead of the ship as it decelerates. Travelling at a constant relativistic speed, the ground up fuel tanks will punch their way through any interstellar dust particles ahead of the ship, clearing a path.

Some dust will still enter the path ahead of the ship, so in addition to that trick, the Valkyrie extends many layers of ‘blankets’ of extremely thin material (’similar to Mylar’) ahead of the ship. Because the ship is accelerating in the opposite direction, the blankets are stretched out in front of the ship.

Obviously, nothing remotely close to this has ever been tried! So who knows if it would actually work. I haven’t seen discussion of relativistic dust in other designs on Atomic Rockets, though.

6. you also need to be safe from the ordinary kind of space radiation

Even if you’re not travelling at relativistic speeds, you will (without the protection of an atmosphere) constantly be irradiated by cosmic rays and other ionising radiation from any nearby star. This is a problem even for current astronauts, who apparently hide behind water tanks during periods particularly intense radiation. And it’s often discussed as a major danger of a voyage to Mars.

To an extent, you are protected from this by the hull of your spaceship, which absorbs most kinds of radiation even if it’s quite thin. You apparently need two layers to deal with different kinds of radiation, since charged particles entering the e.g. lead you would use for gamma ray shields can create deadly X-rays. Everywhere where people will be living, long-term, needs to be protected this way.

During periods of major radiation, such as solar flares, this may not be enough. You need a particularly well-shielded part of your spaceship to hide in at these times. One method is to use the water (that you’re carrying anyway, for the crew to drink) to protect the crew at these times. You need a way for the crew to access this area, and take anything particularly vulnerable to radiation - such as food - in there with them.

Apparently people are also considering other methods, involving things like powerful electric or magnetic fields, or bubbles of plasma. I don’t know a lot about this.

7. you also need to be safe from the radiation from your ship’s propulsion and power plant

To travel long distances in space, chemical rockets tend not really to be enough - you need something with a very high \(\Delta v\). Many such propulsion systems have been designed, but they tend to have the side effect of producing lots of ionising radiation.

This isn’t necessarily a huge problem. You don’t necessarily need to shield your entire reactor (which would be very heavy), but can instead use a shadow shield, like we discussed on the Valkyrie, to keep just your ship safe from the radiation, and let the rest of the radiation radiate away into space. But it does imply that the rest of the ship, including your thermal radiators, needs to keep itself inside the cone of safety created your shadow shield. And your shadow shield is usually still quite massive, because the main way we attenuate radiation is to make a lump of very dense materials.

Here’s an example of a NASA design which uses a shadow shield, and has to keep its radiatiors inside the shadow:

8. it needs to have pressurised areas for humans to breathe in

The pressure outside the spaceship is nothing. The pressure inside the spaceship is probably about 1 atmosphere. Your ship needs to be strong enough to contain that.

You definitely don’t want to have too many parts of your ship that would be particularly vulnerable to rupturing, so the walls of your pressurised areas are probably going to be nice and curved with minimal sharp corners.

One way to build a pressure vessel, the way we currently do, is to use lots of strong metal. But that adds a lot to your payload mass. So another possibility is to have an inflatable spaceship.

NASA has one such design called the TransHab. Which I assume means it can’t be boarded by cis people. It looks like this (source)…

(all of those weird CGI people are trans. no arguing)

arborine has generally talked about much larger, long-term designs: a big sphere or torus, containing a much more comfortable landscape built for many people (I will have to ask her to help me recreate the details though!). Spheres and toruses are good for inflatable shapes, because the tension is nice and even across the surface (well, this is complicated by rotation, but basically).

The need to have a pressurised hull and radiation shielding does introduce some limitations. Windows in a pressurised compartment generally need to be quite small and rounded, like aeroplane cabin windows, but when you’re far from a planet, there’s not really anything to see anyway. (It seems that, with sufficient engineering, you can make bigger windows, such as in the ISS’s observation bubble thingy. Even then, the bubble’s pretty small, and there’s plenty of structural stuff keeping that together.)

9. you need to be able to deal with problems with mimimal damage

Despite all your best efforts, it seems inevitable that a pressurised compartment travelling through the vacuum of space might have something go terribly wrong. For example, something punches a hole in the hull. You need to have that not end the mission, and cause as little harm to the crew as possible.

This probably means, internally, that your ship needs to be divided into separate compartments, so that if one gets depressurised, the rest of the ship is still safe. If the walls of your ship can somehow be self-sealing, even better.

10. you need self-repair and a self-renewing ecosystem for a long voyage

If you’re travelling on a very long voyage, carrying supplies for the entire journey just takes too much mass. You need to carry with you a functional, lasting ecosystem of plants etc. that won’t die off during the voyage.

Making a closed ecosystem is very hard - as far as I know, we haven’t succeeded yet. I will defer to arborine on the details of one, as I can’t remember what she said or what was in her extremely practical space habitats book off the top of my head.

In addition, parts of your ship will slowly fail and go wrong. Atomic Rockets claims the best NASA probes can last 40 years in space, and that may well not be enough for your purposes (even taking as much advantage as possible of relativistic time dilation). For a very long voyage like a generation ship or something, everything can probably be assumed to go wrong your spaceship to an extent needs to have the capabilities on board to build itself, and apply its repairs outside the safety of crew compartments.

Needing to recreate them en route might constrain the materials you can use, and hence the shapes you can make.

( breathofzephyr made an interesting point that the total computing power, data storage etc. available to the crew of a generation ship will, unless they can build more computers, go down over the course of the mission. which could have interesting social effects…)

11. you will probably need to dedicate a lot of your spaceship to storing reaction mass in suitable containers

A consequence of the rocket equation is that, for every gram (or whatever mass unit) of payload - structural elements, people, food, computers, robots, whatever - you have, you will need a certain number of grams of reaction mass to throw out the back of your rocket.

(The exception is if you are using a propulsion system such as a solar sail or a laser sail where you are propelled by collisions with material that you’re not carrying with you. Or if you’ve figured out a way to get the Bussard Ramjet, or its variants, to be practical!)

The shape of your propellant tanks depends a great deal on what kind of engine you’d use, but basically, you need a lot of pressure vessels somewhere on your ship, likely held separate from the crew modules.

12. it needs to be really really light, as far as possible

The rocket equation is a very demanding thing, and unless you somehow have vast amounts of leftover \(\Delta v\), you want to take every possible opportunity to save on mass. (’loads of leftover \(\Delta v\) is like, you’re using Project Orion to travel through the solar system or something.)

This is why lots of spaceship designs are modules held together by a lightweight framework - you want to minimise heavy structural elements as much as possible.

We’ve talked quite a bit about this already, though!

13. if you provide gravity, you either need a lot of \(\Delta v\) or something has to spin

For a long voyage, your crew’s bones and muscle will atrophy unless they’re given an environment with a source of ‘gravity’ similar to Earth’s.

There are two ways you can do this, which amount to accelerating some or all of your spaceship. One way is to have your engine burning so as to provide \(1 g\) acceleration throughout your voyage. By contrast, current space missions usually involve very short rocket burns, with the spaceship flying freely for most of its mission. To burn throughout the mission, your ship will need vastly more \(\Delta v\), and hence vastly more propellant mass per unit payload mass.

Since that’s pretty demanding, the other option is to have part of your spaceship spinning. This could be for example a toroidal ring, or a cylinder, or a sphere, or a couple of rooms on the end of long arms.

Depending on how large your rotating section is, it will have to spin more or less fast in order to maintain the amount of gravity you need. The centrifugal acceleration in a rotating reference frame is given by \(r\omega^2\), where \(r\) is the distance from the centre, and \(\omega\) is the angular speed.

Note this means the artificial ‘gravity’ in a rotating section with multiple floors, or in a spherical habitat, will vary as you get closer or further away from the centre! The further out you are, the stronger gravity will be.

A rotating habitat section has some implications about the design of the inside. ‘Level’ surfaces will be curved instead of flat, and the Coriolis effect will apply to deflect trajectories from a straight line - particularly strongly in a small, fast-spinning section.

There are lots of difficulties to overcome if only part of your spaceship is spinning. A major one is the problem of the bearing between the rotating section and the rest of the ship - it needs to be near frictionless, or your spin section will slowly spin down (and your ship will slowly spin up.

Another problem comes from spinning up your spin section without setting the rest of your spaceship rotating in the opposite direction (with possible consequences of precession, and the like). One solution is to have a flywheel inside your ship, but this implies a large mass that’s generally useless. Another is to have multiple counter-rotating spin sections, such that the net angular momentum change can be 0 without spinning the entire ship.

Or, you can just let the entire spaceship spin at once, though this makes manouevring the ship somewhat complicated because of the way gyroscopes act (it becomes forced precession, and you need to push at \(90^\circ\) from the way you actually want to turn…)

A further problem comes from the fact that, when your spaceship is accelerating, the direction of ‘down’ will change. This can be accomodated by some very complicated-looking designs involving arms that swing in and out (source), or just ignored because most of the time your ship is not accelerating.

and more stuff, probably

So, yes, there are a lot of physical demands on the shape of a spaceship, even one that never visits an atmosphere!

Building a spaceship is a lot different from building on Earth, and of course nobody actually knows how to build a spaceship for an interstellar voyage. But this is what we think, based on the physical constraints we’ve imagined so far…

Avatar
Avatar
reblogged

The moon passed between ‪NASA‬’s Deep Space Climate Observatory and the Earth, allowing the satellite to capture this rare image of the moon’s far side in full sunlight. As the moon is tidally locked to the Earth and doesn’t rotate, we only ever see the one face from the Earth. Awesome shot!

This isn’t the only picture that DSCOVR (that is the official acronym, I know…) took of the moon! Here’s a gif of the pictures of the moon transit, from Wikimedia Commons:

This picture makes the Earth and Moon look really close. You can see a to-scale orthographic picture here that shows how far they are apart on average.

The DSCOVR satellite orbits at the Sun-Earth L1 Lagrange point, which allows it to stay more or less directly between the Earth and Sun.

(A Lagrange point is a place in relation to two orbiting bodies where a third small body will orbit in a fixed position relative to them, i.e. an object exactly at the L1 point will stay fixed between the Earth and Sun. There are five Lagrange points, but three of them - the L1 point included - are always unstable, so if something isn’t quite at the Lagrange point, it will move away, slowly at first and faster the further away it gets. The DSCOVR satellite makes use of a trick called a Lissajous orbit, which allows it to effectively orbit the Lagrange point itself, staying fairly near it. But over a long period, this orbit is unstable, and will require corrections.)

The L1 point is about 1.5 million km from Earth. The Moon orbits the Earth with a semi-major axis of 384 thousand km. So on average, the DSCOVR satellite is about four times further away from the Earth than the Moon.

Avatar
reblogged
Avatar
canmom

A time traveller comes to a small deserted town. She lives in this town for a month, then travels back in time one month. She does this again, six hundred times.

To an observer, a traveller arrives, and suddenly - for one month - the town is bustling with the activity of six hundred people. Then, it all shuts down. The traveller continues on, now fifty years older than when she entered.

Avatar
argumate

you can also use this trick to overwhelm an opponent, but make sure they don’t get a hit in because you are vulnerable to an attack on all of your instantiations.

Not necessarily! Only the first couple of instances need to avoid hits. The “older” (traveller pov) versions who have already seen that fight multiple times will definitely have an advantage, knowing how things already transpired.

(Also the hits won’t be too bad if there ARE older versions around…)

Going, as physicists who speculate about time travel often do, by the Novikov self-consistency principle, there cannot be a timeline where someone is rendered unable to time travel in the presence of their future self.

So, if one of the ‘yous’ gets injured in the fight, it will never be enough to make them no longer able time travel. The injury will afflict every future copy, of course.

So if you’re about to fight someone and five copies of you appear and they all have the same injury that you don’t, you will know that you’re about to suffer that injury in the fight. This knowledge will not allow you to avoid the injury! All your future copies knew about the injury as well, and still suffered it.

If say 2/5 of your future copies have the injury, you’ll know that you’ll suffer the injury on your fourth loop through the fight.

You might also see one of your copies die (implicitly, the final copy). In this case, despite knowing it will prove fatal, you will necessarily still time travel through the fight several times before dying. Likewise if you see one of your copies become injured so they can’t time travel: that one is the final copy, and you can look forward to that.

You won’t ever see two copies of yourself die, because that would be inconsistent. In a natural-seeming way, the timeline will simply never include such an event, despite anyone’s best efforts.

What happens if i go back in time to try and change a thing, does the time machine just not work if that would happen were i to go back? 

either your attempt to change the thing is already part of the timeline, and unsuccessful, or else yes, the time travel doesn’t work

novikov self-consistency says only that the timeline is consistent - it doesn’t say anything about how that consistency is maintained. it might be that time travel could only work in tightly controlled conditions to ensure consistency. which could still allow you to implement time travel based algorithms!

objection

not a literal objection just me being difficult but 

novikov self-consistency is a conjecture – nay, a coping mechanism – to help physicists who don’t wanna go “but what if causality is arbitrary”

have we ever actually demonstrated it? maybe we only think causality works because it’s like gravity, and we’re still figuring out airplanes and spaceships

what does this scenario look like if the time traveller isn’t bound by closed loops

what now science side of tumblr

oh, absolutely. there’s no evidence of novikov self-consistency - it’s just an option for when you say “ok, suppose imagine we allow time travel and throw out causality, what rule could we have?”

replacing novikov self-consistency with causality allows for all sorts of weird shit, too! at any moment, an object inside a stable time loop could appear.

so while we tend to speculate along the lines of ‘what if you could meet yourself time travelling from the future’, you might also bump into an automatic time machine that only exists inside a time loop.

What rule would govern when time travellers spontaneously appear? Usually I think we say ‘while what if the conservation laws - energy, momentum and so on - are more fundamental than causality, and apply even to time travellers’.

This is where my questionable analogy with Feynman diagrams comes in. But I’ll elaborate on that below a readmore…

So you might imagine - a time traveller can spontaneously appear, but only as long as all the relevant conservation laws are maintained. Perhaps then to achieve significant time travel you need a ‘receiver’ time machine that produces a large amount of energy that turns into a time traveller, and a ‘sender’ time traveller that (to outside observers) disintegrates a time traveller.

That still doesn’t include anything at all getting out of the ‘time machine’, as long as it gets back in the time machine later.

Novikov self-consistency can’t be the only way to think about time travel, but we do need some handle for our speculations. So without Novikov, we need some other way to constrain what happens… I don’t know what that would be.

Avatar
reblogged

Real quick question for a different kind of scientist. How quickly and easily do you think someone could construct a device capable of flinging my body into the sun?

To be clear,

Do you think if I fling myself into the sun I’ll have to write this final tomorrow?

Well, if you’re not too concerned about surviving, you could, in principle, just use a really big bomb. But I doubt you’d be able to scrounge up the parts, and you wouldn’t make it all the way to the sun by tomorrow.

So we’ll assume that the sun is the only safe place in the universe where you will not be asked to complete the final. If so, you need to reach the sun within a day.

The sun is small compared to the distance between Earth and Sun, so the minimum average speed you’ll need is about 1AU/1day, which is 1700km/s or about 0.6% of the speed of light.

In general, just pointing your rocket at the sun and firing for that much delta-v won’t get you to the sun, because it will instead perturb you onto a very elliptical orbit that will swing past the sun fairly close before orbiting on a wide arc that will leave you far too much time to complete your final. So you need to fire in a way that cancels the Earth’s orbital speed in addition to increasing your velocity in a Sun-wards direction, increasing the total delta-v budget.

That said, the Earth’s orbital speed is “only” about 30km/s, which is very small compared to what we need to reach the sun on time. So we’ll ignore this correction.

As you approach the sun, you will gain additional speed because of gravitational acceleration, but let’s assume we want to achieve that minimum speed using a rocket, bomb, etc. And we want to do it quickly: if the thrust is to low, even if you eventually achieve that speed, you’ll do it too late.

Basically what I’m saying is we probably need to resort to something like Project Orion, or antimatter propulsion, or else an extremely powerful railgun. I’ll come back to this later.

So checking Wikipedia’s article on Project Orion - the nuclear bomb pushed rocket design - it seems that Freeman Dyson suggested a ‘momentum limited’ variant of the design that could accelerate at 1g up to a delta-v of 10,000 km/s (over the course of ten days), with the rocket itself weighing fifty million kg empty, and carrying a fifty million kg structure and payload. (The nuclear bombs make up another 300 million kg.)

This spaceship could not be built in a day; with present technology it could not even be built at all. But, if it had already been built, the first day’s acceleration would get you about 1000 km/s, not quite enough to meet your final deadline. At least, if you had the full 50 gigagram structure on board. But you only need to be on the spaceship for a day! By dropping everything but basic life support (or even that), we could double the acceleration, and quadruple the final speed, which should get us to the sun on time.

Dyson estimated the cost of this machine to be $0.367 trillion, or 10% of the entire US annual GDP.

We’re also ignoring the difficulties of getting you into orbit in the first place, which would make a difference to the delta-v budget - but not a big one, considering, as the Earth escape velocity is “only” 11km/s.

Building Project Orion is of course currently outside our technical capabilities, not to mention banned by international nuclear testing laws. The same goes for massive railguns and antimatter. Could we instead use any past or currently existing rockets to get you en route to the sun, even if you won’t get there in a day?

A payload-less Saturn V has a delta-v of about 18km/s. Your mass is probably negligible compared to that of the rocket, so this applies. But that is not enough to cancel the Earth’s 30 km/s orbital speed and reach the sun. That said, the Saturn V’s first two stages are designed to operate in the Earth’s atmosphere, so you could get better efficiency by using rockets more like its third stage.

I’m not sure any mission has ever gone particularly close to the sun, for the straightforward reason that if you get to close to the sun you will be vaporised by incident radiation. But we have sent spacecraft to Mercury, which has similarly extreme delta-v requirements. The Mariner 10 probe made flybys of Mercury, making use of a gravitational slingshot around Venus to dump excess velocity. The MESSENGER probe, after several flybys of Earth and Venus to adjust its trajectory gravitationally, was able to enter the orbit of Mercury after being launched with a Delta II rocket.

So your best bet for reaching the sun would be to use Venus, and indeed Mercury, for gravity assists. Working out a suitable trajectory is a bit beyond me at the moment!

Avatar
reblogged
Avatar
canmom

So Keystone Motel was an utterly fantastic episode (& what an amazing bonus that it aired on my birthday!), and shortly after I watched it I thought of something: given the assumed volume of the pool and the presumed time it took for our tiny gay space rock's eternal flame to evaporate it, can we calculate how hot she'd have to be for how long to completely boil away all of that water?

Avatar

The short answer is of course exactly artistic license degrees fiction.

But if you really want a calculation, well, we can do one.

The first thing we need is the volume of the motel pool. It looks like a pretty small pool:

But also surprisingly deep, since Steven and Ruby are both out of their depth.

I noticed a weird thing here: the steps don’t go all the way to the bottom of the pool. Ruby is only about chin deep when she reaches the bottom of the steps, but she keeps going down underwater.

If we assume Steven and Ruby are the height of an average 12-year-old child, which Wolfram Alpha informs me is about 1.5m, then the pool is about 1.5-1.7m deep. Pretty deep for a hotel pool!

Now, the next question is the area. According to the establishing shot of the motel, it’s shaped a bit like a kidney bean.

Though it’s in perspective, this is a very long shot and we can act as if it’s orthographic. Which means we can use Greg’s van as a measuring-stick to estimate its area.

The pool appears to be about three and a bit van-lengths in one direction…

and about five and a half van widths in the other.

I don’t know much about vans, but Wikipedia advises me a common kind of van is the Ford Transit. Whatever Greg’s van is, it’s probably similarly sized. Wolfram Alpha tells me a Ford Transit is about 3.5m long and 2.1m wide. So that places our pool as being bounded by a box roughly 11.5m long on each side.

Now, the pool is not rectangular, so we need to work out how much of this box is taken up by the pool.

In ‘screen space’ (to use a graphics term), the pool takes up about 60% of the area of the bounding box. If we can assme the perspective transformation is roughly linear, we can estimate that the actual area of the pool is also 60% of the physical area of the bounding box. With this assumption, the pool is about 80 square metres in area.

This means the total volume of the pool is about 130 cubic metres. Awesome. All we need to work out the energy needed to vaporise it is an estimate of its temperature!

Judging by the lighting, and the fact that Steven was quite happy to jump right in without comment, it’s probably a heated pool. Heated recreational pools are apparently supposed to be kept no hotter than 29°C, so for a lower bound, let’s assume that temperature as the starting point before Ruby gets in.

So the total energy Ruby must supply to vaporise the entire pool is $$\Delta E = V\rho (c \Delta T + \Delta h_v)$$where \(V\) is the volume of the pool, \(\rho\) is the density of water (thus \(V\rho\) is the mass of water in the pool), \(c\) is the specific heat capacity of water, \(\Delta T\) is the temperature change required to raise the water to its boiling point of \(100 \unit{°C}\), and \(\Delta h_v\) is the specific enthalpy of vaporisation (also called the latent heat of vaporisation).

Plugging in numbers to that calculation (which unfortunately Wolfram Alpha’s natural-language input doesn’t seem to understand directly so I had to copy values in) gives a value of 330GJ to vaporise the entire pool. This is about 91 MWh. For comparison, the Hinkley Point B nuclear power station near where I live currently generates about 485-500MW electricity per reactor (which is about 80% capacity), so you could produce enough energy to vaporise the pool by using all the energy from one reactor for about ten minutes.

OK, but you also wanted to know the average power Ruby was producing! For this we need an estimate of the time. Ruby started to boil the pool within seconds of getting into it (she steps in at 4:54 and Steven comments on the warm spot around 5:00, and the pool starts to boil at around 5:05). The bubble volume looks significant, but it’s hard to tell how much water she’s actually boiled at this point.

By 6:59 in the episode, the pool has completely evaporated. I can’t see much evidence of a time skip between these points, but then this implies Greg is only away for a few minutes, so it’s probably artistic license again. (There might be a time skip between Steven leaving to pee and Greg’s return I guess.)

Greg asked them to call the police if he was away for an hour, so getting the roller probably only took tens of minutes at most.

The result is, basically, Ruby generates between a couple of hundred megawatts and a couple of gigawatts of power. (I’m not going to try and analyse the complications of heat transfer to the water here! But it would slow the process down.) In other words, an angry Ruby emits about as much power as a nuclear reactor or power station.

This would probably have other effects - Ruby would also heat the air around her, and quite probably quite quickly set the motel room on fire if she stayed in there for long. So it’s pretty lucky Steven invited her into the pool, really! At least the pool’s ceramic tiles are heat resistant. The enormous cloud of hot steam wafting out of the pool, however, would probably cause scalding and injuries if anyone walked into it.

(In terms of industrial uses, Sapphire’s freezing powers might be more useful, because it’s a lot easier to make things hotter than to cool them down!)

Avatar
You are using an unsupported browser and things might not work as intended. Please make sure you're using the latest version of Chrome, Firefox, Safari, or Edge.